Você está na página 1de 40

FÍSICA FRENTE B

Antônio Máximo
Beatriz Alvarenga
Carla Guimarães

CONSERVAÇÃO DA QUANTIDADE DE MOVIMENTO


1 Conservação da quantidade de movimento . . . . . . . 04
Impulso e quantidade de movimento . . . . . . . . . . . . . . . 04
Quantidade de movimento de um sistema de partículas . 08
Conservação da quantidade de movimento . . . . . . . . . . 12
Colisões . . . . . . . . . . . . . . . . . . . . . . . . . . . . . . . . . . . 17
Revisão . . . . . . . . . . . . . . . . . . . . . . . . . . . . . . . . . . . 30

2121656 (PR)
MÓDULO
Conservação da quantidade
de movimento

Jesse van Bezooijen, do time NAC Breda, comete


falta em Alex Schalk, do Go Ahead Eagles, em partida do
Campeonato Neerlandês de Futebol (Eredivisie), a mais
importante liga de futebol dos Países Baixos. Deventer,
Holanda, 22 de novembro de 2014.
VI IMAGES/GETTY IMAGES
REFLETINDO SOBRE A IMAGEM

A conservação da quantidade de movimen-


to é uma lei de aplicação muito ampla, cons-
tituindo, por isso, um poderoso recurso usado
pelos cientistas para estudar os fenômenos
naturais. O que acontece quando uma bola de
futebol está em contato com o pé de um jo-
gador? Se, em uma partida de futebol, um dos
jogadores cometer uma falta derrubando outro
jogador, será pior para o jogador que caiu se a
falta for cometida por um jogador magro de
massa m, mas muito veloz, ou se a falta for co-
metida por um jogador com o dobro da massa
1
(2m) mas com da velocidade?
4
Mesmo em situações complexas como estas,
observa-se que há conservação da quantidade
de movimento.

www.sesieducacao.com.br
CAPÍTULO

1 Conservação da quantidade
de movimento

Veja, no Guia do Professor, o quadro de competências e habilidades desenvolvidas neste módulo.

Objetivos:
IMPULSO E QUANTIDADE DE MOVIMENTO
c Reconhecer os A lei da conservação da energia propõe que a energia não pode ser criada nem destruída, apenas
conceitos de impulso transformada. Essa lei é de enorme importância no campo da Física e seu uso facilita a solução de
e sua relação com inúmeros problemas.
a quantidade de Existem outras leis de conservação na natureza que envolvem outras grandezas, que também
movimento. se conservam, em determinadas circunstâncias.
c Identificar as forças Uma dessas leis, a conservação da quantidade de movimento, será analisada neste módulo.
que atuam e influem O conceito de impulso e sua relação com a quantidade de movimento constituem o ponto de
na quantidade de partida para chegarmos a essa lei de conservação. Por isso, iniciaremos com a introdução desses
movimento de um conceitos.
sistema de partículas.
O que é impulso
c Aplicar o princípio Quando um jogador de futebol cobra uma penalidade ou quando um tenista, usando a sua
da conservação raquete, rebate a bola, há uma força que atua durante um curto intervalo de tempo sobre a bola, o
da quantidade de que faz com que ela seja impulsionada.
movimento. De um modo geral, sempre que uma força atuar em um corpo durante um certo intervalo de tempo,
diremos que o corpo recebeu um impulso. Para o caso de uma força F& constante, atuando durante um
intervalo de tempo Δt (fig. 1), define-se o impulso I, & exercido pela força, por meio da expressão:

I &5 F & ? Δt
I&
t1 t2 Observe que I & é um vetor que tem a mesma direção e o mesmo sentido de F&, como mostra a
F& F& figura 1. No Sistema Internacional (SI), a unidade de impulso é N ? s (equivalente a kg ? m/s).
I & 5 F& ? Δt

Δt 5 t2 2 t1 Forças impulsivas

CHEN WS/SHUTTERSTOCK/GLOW IMAGES


Quando uma bomba explode ou quando
Fig. 1 – A força, atuando no corpo, exerce dois automóveis colidem, e em várias outras
nele um impulso.
situações semelhantes, ocorrem, entre os corpos
que participam desses fenômenos, forças muito
grandes, mas que atuam durante um intervalo
de tempo muito curto. Por exemplo, quando um
jogador de futebol rebate uma bola, a força de
interação entre a bola e o pé do jogador é da
ordem de 103 kgf e dura cerca de 0,01 s. Essas
forças são denominadas forças impulsivas.
Deve-se observar que essas forças, em geral,
provocam grandes acelerações nos corpos
em que atuam, isto é, atuando em intervalos
de tempo muito curtos, provocam variações
apreciáveis nas velocidades desses corpos.

Uma força impulsiva tem um módulo muito grande e


atua durante um intervalo de tempo muito pequeno.

4 Conservação da quantidade de movimento


Quantidade de movimento
A figura 2 mostra um corpo de massa m movendo-se com uma velocidade v &. Uma grandeza
muito importante, relacionada com o movimento do corpo, é a sua quantidade de movimento.
Essa grandeza, também denominada momento linear do corpo, e que vamos representar por q &, é
definida da seguinte maneira:

A quantidade de movimento (ou momento linear), q ,& de um corpo de massa m, que se v& q& 5 m v &
move com uma velocidade v &, é definida pela expressão:
m

q & 5 mv & Fig. 2 – Uma partícula de massa m, com


velocidade v ,& possui uma quantidade de
movimento q & 5 mv .& A quantidade de
A quantidade de movimento é uma grandeza vetorial, de mesma direção e mesmo sentido movimento e a velocidade têm a mesma
do vetor v &, como mostra a figura 2. No SI, a unidade de quantidade de movimento é kg ? m/s (não direção e o mesmo sentido.
recebe denominação especial).

Relação entre impulso e quantidade de movimento


Consideremos um corpo de massa m movendo-se com uma velocidade v &1. Se uma força F&
constante atuar no corpo durante um intervalo de tempo Δt, sua velocidade sofrerá uma variação,
passando a ser v 2& final desse intervalo (fig. 3).
F& F&
v &1 t1 v 2& t2

CASA DE TIPOS/ARQUIVO
DA EDITORA
Linha de
chegada

Impulso Aceleração

Fig. 3 – O impulso de uma força sobre o corpo provoca uma variação em sua quantidade de movimento, ou
seja, o impulso modifica a quantidade de movimento. Quanto maior for o impulso, maior será a velocidade v&2
em comparação com a velocidade v1& .

Supondo que F& seja a resultante das forças que atuam no corpo, a segunda lei de Newton nos
permite escrever que:
F& 5 ma & COMENTÁRIO
 Veja a figura abaixo. Nessa si-
 Δv
em que a & representa a aceleração adquirida pelo corpo e a 5 . Logo: tuação, podemos dizer que será
Δt
tanto mais difícil fazer um veí-
culo parar quanto maior for sua
m ? Δv &
F &5 ∴ F &? Δt 5 m ? Δv & quantidade de movimento, isto
Δt é, quanto maior for o produto de
sua massa por sua velocidade.
Como a variação da velocidade é Δv & 5 v 2& 2 v 1& , temos:
F & ? Δt 5 m ? ( v 2& 2 v 1& ) ou F& ? Δt 5 mv 2& 2 mv 1&
70 km/h
CASA DE TIPOS/ARQUIVO
DA EDITORA
Observemos, entretanto, que:
F& ? Δt representa o impulso I & que o corpo recebeu;
mv 2& representa a quantidade de movimento do corpo, q&2, no fim do intervalo Δt; FRENTE B

mv 1& representa a quantidade de movimento do corpo, q&1, no início do intervalo Δt.


Assim: 70 km/h
I & 5 q&2 2 q&1 ou I & 5 Δq&
FÍSICA

Quanto maior for a massa,


Portanto, chegamos à conclusão de que o impulso recebido pelo corpo é igual à variação maior será a quantidade de
movimento que um veículo
de sua quantidade de movimento (teorema do impulso). Apesar de ter sido demonstrado para deverá empregar para fazê-lo
o caso de uma força constante, este resultado é geral, isto é, em qualquer situação podemos parar.
afirmar que:

Conservação da quantidade de movimento 5


O impulso, I &, exercido pela resultante das forças que atuam sobre um corpo, durante
certo intervalo de tempo, é igual à variação da quantidade de movimento, Δq&, ocorrida naquele
intervalo de tempo, isto é:
I & 5 Δq & 5 q 2& 2 q1&

A relação entre o impulso e a variação da quantidade de movimento é semelhante à relação


entre o trabalho e a variação da energia cinética (τAB 5 εcB 2 εcA)

EXERCÍCIOS RESOLVIDOS

1 A resultante das forças que atuam no corpo da figura 3 vale F 5 4,0 N e atua durante um intervalo de tempo Δt 5 6,0 s.
a) Qual é o impulso recebido pelo corpo?
b) Se a quantidade de movimento inicial do corpo era q1 5 16 kg ? m/s, qual será o seu valor no fim do intervalo de tempo
considerado?
RESOLUÇÃO:
a) O valor do impulso é dado por:
I 5 F ? Δt 5 4,0 ? 6,0 [ I 5 24 N ? s
A direção e o sentido de I & são os mesmos da força F&.
b) Sabemos que a variação da quantidade de movimento do corpo é igual ao impulso que ele recebeu, isto é:
Δq 5 I [ Δq 5 24 kg ? m/s
Mas Δq & 5 q &2 2 q &1 [ q &2 5 q &1 1 Δq &.
Como a partícula se desloca em linha reta (fig. 3), os vetores q &2, q &1 e Δq & têm a mesma direção. Logo:
q2 5 q1 1 Δq 5 16 1 24 [ q2 5 40 kg ? m/s

2 Uma bola de tênis, de massa m 5 100 g e velocidade v1 5 10 m/s, é rebatida por um jogador, retornando com uma velocidade v 2&
de mesmo valor e direção que v 1& , porém de sentido contrário.
a) Qual foi a variação da quantidade de movimento da bola?
b) Supondo que o tempo de contato da bola com a raquete foi de Δt 5 0,01 s, qual foi o valor da força (considerada constante)
que a raquete exerceu sobre a bola?
RESOLUÇÃO:
a) No instante em que a bola atinge a raquete, o valor de sua quantidade de movimento é:
q1 5 mv1 5 0,100 ? 10 [ q1 5 1,0 kg ? m/s
No instante em que ela abandona a raquete, sua quantidade de movimento vale:
q2 5 mv2 5 0,100 ? 10 [ q2 5 1,0 kg ? m/s
Os vetores q 2& e q1& têm a mesma direção, mas sentidos contrários. Portanto, a quantidade de movimento da bola variou
de 1,0 kg ? m/s em um sentido para 1,0 kg ? m/s em sentido contrário. Quando isso ocorre, devemos atribuir sinais a es-
ses valores, considerando, por exemplo, negativo o sentido inicial do movimento e positivo o sentido contrário. Nessas
condições, a quantidade de movimento variou de 21,0 kg ? m/s para 11,0 kg ? m/s, isto é, a variação da quantidade de
movimento da bola foi:
Δq 5 q2 2 q1 5 1,0 2 (21,0) [ Δq 5 2,0 kg ? m/s
b) O impulso I & 5 F& ? Δt que a raquete exerceu na bola é igual a Δq &, isto é:
∆q 2,0
F ? ∆t 5 ∆q ∴ F 5 5
∆t 0,01
F 5 2,0 ? 102 N

6 Conservação da quantidade de movimento


As competências e habilidades do Enem estão indicadas em questões diversas ao longo do módulo. Se necessário, explique aos alunos que a utilidade deste
“selo” é indicar o número da(s) competência(s) e habilidade(s) abordada(s) na questão, cuja área de conhecimento está diferenciada por cores (Linguagens: la-
ranja; Ciências da Natureza: verde; Ciências Humanas: rosa; Matemática: azul). A tabela para consulta da Matriz de Referência do Enem está disponível no portal.

PARA CONSTRUIR
CA
SA
1 O bloco mostrado na figura deste exercício está se deslocan- 3 (UFTM-MG) Num trecho DE
TIP
OS
vF /A
do, em movimento retilíneo, sob a ação de uma força resul- m plano e horizontal de RQ
Ene-5
UI
VO
C 2
tante F 5 5,0 N. A força F& atua desde o instante t1 5 2,0 s até H-2 uma estrada, um carro DA
ED
ITO
o instante t2 5 6,0 s. m
faz uma curva mantendo RA
Ene-6
C 0
H-2
constante o módulo da sua

AVITS ESTÚDIO
GRÁFICO/
ARQUIVO DA
EDITORA
v1& F& v2& velocidade em 25 m/s. A figura
mostra o carro em duas posições, 120°
t1 5 2 s t2 5 6 s
movendo-se em direções que fazem, entre
si, um ângulo de 120°.
a) Qual é o valor do impulso I & produzido pela força sobre o
v0
bloco? Considerando a massa do carro igual a
Observando que a força F& atuou durante um intervalo de tempo 1 000 kg, pode-se afirmar que, entre as duas
Δt 5 t2 2 t1 5 4,0 s, o valor do impulso I & exercido sobre o bloco será: posições indicadas, o módulo da variação
I 5 F ? Δt 5 5,0 ? 4,0 [ I 5 20 N ? s da quantidade de movimento do veículo,
b) Desenhe na figura o vetor I &. Desenhe, também, a variação em (kg?m)/s, é igual a: b
da quantidade de movimento Δq& que o impulso provo- a) 10 000.
cou no bloco. Veja comentários sobre este item no Guia do
Professor. b) 25 000.
c) Suponha que o valor da quantidade de movimento do blo- c) 12 500.
co, no instante t1, fosse q1 5 10 kg ? m/s. Desenhe, na figura, d) 12 500 2.
o q&1. e) 25 000 2.
d) Lembrando-se de sua resposta ao item a, determine o va- A mudança do sentido do veículo implica variação da velocidade.
lor de q&2. Logo, analisando os vetores da quantidade de movimento, temos:
Antes da curva:
Δq& 5 q&2 2 q&1 ⇒ q&2 5 q&1 1 Δq&

Ao traçar os vetores q&1 e Δq&, o estudante perceberá que eles têm


a mesma direção e o mesmo sentido. Assim, a soma vetorial v &0
q&1 1 Δq& obtida por meio de uma simples soma algébrica, isto é q&0 5 vetor quantidade de movimento inicial
(lembrando-se que Δq 5 20 kg ? m/s):
q2 5 q1 1 Δq 5 10 1 20 [ q2 5 30 kg ? m/s

Depois da curva:
2 Considere um corpo se deslocando em movimento retilíneo q&F 5 vetor quantidade de movimento final
m uniforme. Inicial
Ene-4
C 5
H-1 a) A quantidade de movimento desse corpo está variando? v&F
Explique.
A velocidade do corpo permanece constante em módulo, direção
Combinando os dois vetores:
e sentido. Logo, a quantidade de movimento (q & 5 mv&) deste corpo
Δq& 5 q&F 2 q&0
não está variando.
Δq&
b) Tendo em vista a resposta do item anterior, o que você
conclui sobre o impulso que atua no corpo?
q&0
Vimos que I & 5 Δq&. Como q& é constante, temos Δq & 5 0 e,
60°
então, I & 5 0 (não há impulso resultante sobre o corpo). q&F
FRENTE B
c) Qual é o valor da resultante das forças aplicadas no corpo? 120º

Se não há impulso sobre o corpo, concluímos que é nula a


resultante das forças que nele atuam (temos I & 5 F& ? Δt e como O ângulo formado entre o vetor q&0 e q&F é 60° e |q&0| 5 |q&F|, logo o
FÍSICA

I & 5 0, obtemos F& 5 0). Este resultado está em concordância triângulo formado pelos vetores q&0 e q&F é equilátero.
com a primeira lei de Newton, pois, se o corpo está se deslocando | Δ q& | 5 m ? | v &0 | ⇒ | Δ q& | 5 1000 ? 25 ⇒

em movimento retilíneo uniforme, deve ser nula a resultante das ⇒ | Δ q& | 5 25 000 kg ? m/s

forças que atuam sobre ele.

Conservação da quantidade de movimento 7


4 (Vunesp) O gol da conquista do tetracampeonato pela Admita que, nessa jogada, a bola ficou em contato com o
m Alemanha na Copa do Mundo de 2014 foi feito pelo peito do jogador por 0,2 s e que, nesse intervalo de tempo,
Ene-6
C 4
H-2 jogador Götze. Nessa jogada, ele recebeu um cruzamento, a intensidade da força resultante (FR), que atuou sobre ela,
m
matou a bola no peito, amortecendo-a, e chutou de variou em função do tempo, conforme o gráfico.
Ene-6
C 5
H-2
esquerda para fazer o gol. Considere que, imediatamente
antes de tocar o jogador, a bola tinha velocidade de FR (N) Δq& 5 mv &2 2 mv1& ⇒ Δq & 5 0, 4 ? (20, 6) 2 0, 4 ? 8 ⇒
m
Ene-5
C 7 módulo v1 5 8 m/s em uma direção perpendicular ao seu ⇒ Δq & 5 20, 24 2 3,2
2 ⇒ Δq & 5 3, 44 kg ? m/s
H-1
peito e que, imediatamente depois de tocar o jogador, Fmáx
sua velocidade manteve-se perpendicular ao peito do
jogador, porém com módulo v2 5 0,6 m/s e em sentido
contrário.
Antes Depois
t (s)
0 0,2
v &2

v&1 Considerando a massa da bola igual a 0,4 kg, é correto afirmar


que, nessa jogada, o módulo da força resultante máxima que

CASA DE TIPOS/ARQUIVO DA EDITORA


atuou sobre a bola, indicada no gráfico por Fmáx , é igual, em
newtons, a: b
a) 68,8. A área sob o gráfico é igual ao impulso da
b) 34,4. força resultante, logo:
c) 59,2. F ? ∆t
I & 5 ∆q& ⇒ máx 5 ∆q ⇒
d) 26,4. 2
e) 88,8. ⇒ Fmáx 5 2∆∆t q ⇒ Fmáx 5 2 ?0,2
3,44
⇒ Fmáx 5 34,4 N

TAREFA PARA CASA: Para praticar: 1 a 3 Para aprimorar: 1 e 2

QUANTIDADE DE MOVIMENTO DE UM SISTEMA


DE PARTÍCULAS
Quantidade de movimento total
A figura 4 representa um sistema de partículas de massas m1, m2, m3, etc., que se movem com
velocidades v1& , v2& , v3& , etc. As quantidades de movimento de cada partícula serão:
q1& 5 m1v1& , q2& 5 m2v2& , q3& 5 m3v3& , etc.
A quantidade de movimento do sistema, ou seja, a sua quantidade de movimento total, Q&,
será obtida pela soma vetorial das quantidades de movimento das partículas do sistema, isto é, Q&
é a resultante das quantidades de movimento q1& , q2& , q3& , etc.
Portanto:
Q& 5 q1& 1 q2& 1 q3& 1 … ou Q& 5 Sq &
Logo, para obter Q&, você deverá usar os conhecimentos sobre adição de vetores.

m1

v1&
v2&
m2

Fig. 4 – A quantidade de v3&


movimento total de um sistema m3
de partículas é igual à resultante
das quantidades de movimento
de cada partícula.

8 Conservação da quantidade de movimento


EXERCÍCIO RESOLVIDO

3 Em uma mesa de sinuca, três bolas, cada uma com 0,50 kg de RESOLUÇÃO:
massa, estão em movimento com velocidades v &1 , v &2 e v &3 , O valor do módulo da quantidade de movimento de cada
como mostra a figura abaixo. Sabendo-se que, em um dado bola é:
instante, v1 5 2,0 m/s, v2 5 1,0 m/s e v3 5 2,0 m/s, determine a q1 5 m1v1 5 0,50 ? 2,0 [ q1 5 1,0 kg ? m/s;
quantidade de movimento total do sistema constituído pelas q2 5 m2v2 5 0,50 ? 1,0 [ q2 5 0,50 kg ? m/s;
bolas nesse instante.
q3 5 m3v3 5 0,50 ? 2,0 [ q3 5 1,0 kg ? m/s.

q&1

Q&
q&'

v1&
q&3

(1)

q&2

v3&
Os vetores q&1, q&2 e q&3 estão representados no diagrama aci-
(3)
ma. Os vetores q&1 e q&2 têm a mesma direção e sentidos con-
trários. Sua resultante, q&' 5 q&1 1 q&2, tem um módulo igual à
(2) diferença entre os módulos q&1 e q&2, isto é, o vetor q&' mostrado
AVITS ESTÚDIO GRÁFICO/
ARQUIVO DA EDITORA

na figura acima tem módulo q' 5 0,50 kg ? m/s.


A quantidade de movimento total Q& será dada pela resultan-
v2&
te de q&' e q&3. Como esses vetores são perpendiculares entre
si, podemos escrever:
Q 2 5 (q')2 1 (q3)2 5 (0,50)2 1 (1,0)2 [ Q . 1,1 kg ? m/s.
A direção e o sentido de Q& estão representados na figura.

Forças internas e externas


As forças que atuam em um sistema de partículas podem ser classificadas como forças
internas ou forças externas. Se uma partícula do sistema exercer uma força em outra partícula
que também pertença ao sistema, esta força será uma força interna. Se a força que atua em
uma partícula do sistema for exercida por um agente que não pertença ao sistema, ela será uma
força externa.
FRENTE B
Por exemplo, suponha que tenhamos escolhido um sistema de partículas constituído
apenas pelas bolas branca e preta em uma mesa de sinuca. Dando-se uma tacada na bola
branca, estará atuando, no sistema, uma força externa. Se a bola branca colidir com a bola
preta, a força que atua na bola preta será uma força interna. Já se a bola branca tivesse colidido
FÍSICA

com a bola amarela, a força que ela receberia da bola amarela seria externa, pois o sistema é
constituído apenas pelas bolas branca e preta. Entretanto, se outra pessoa tivesse escolhido
todas as bolas existentes sobre a mesa como seu sistema, as forças entre as bolas branca e
amarela seriam internas a esse sistema. Mas a força exercida pelo taco sobre qualquer uma das
bolas ainda seria uma força externa.

Conservação da quantidade de movimento 9


Forças internas não provocam variação em Q %
Consideremos um sistema no qual uma partícula A exerça uma força sobre outra partícula B
também do sistema (fig. 5). Pela terceira lei de Newton, sabemos que a partícula B reage sobre A com
uma força igual e contrária. Essas forças, como vimos, são forças internas ao sistema. Por causa dessa
interação, a partícula A recebe um impulso I A& enquanto B recebe um impulso I B& . Como as forças que
provocam os impulsos são iguais e contrárias, pois formam um par, ação e reação, e atuam nos dois
corpos durante o mesmo intervalo de tempo, concluímos que: I A& 5 2I B& .

I A& A

I B&
B

Fig. 5 – As forças internas de ação e


reação provocam impulsos de mesmo
I A& 5 2I B&
módulo, mas de sentidos contrários.

Sejam Δq&A e Δq&B as variações nas quantidades de movimento de A e B provocadas por esses
impulsos. Pelo que vimos até o momento, podemos escrever:
I A& 5 Δq&A e I B& 5 Δq&B
Logo:
Δq&A 5 2Δq&B
As forças internas provocarão variações iguais e contrárias nas quantidades de
movimento das partículas do sistema. Como consequência desse resultado, as forças internas
não provocam variação na quantidade de movimento total, Q&, do sistema. De fato, como:
Q& 5 q1& 1 q2& 1 q3& 1 …
se uma força interna provocar uma variação em q&, haverá uma variação igual e contrária na
quantidade de movimento de outra partícula (em q&4 , por exemplo). Essas variações se anularão e a
quantidade de movimento total, Q& , permanecerá invariável.
Chegamos, assim, à seguinte conclusão:

As forças internas podem provocar variações nas quantidades de movimento de cada partícula
de um sistema, mas não provocam variação na quantidade de movimento total do sistema.

PARA CONSTRUIR

5 Considere o sistema constituído pela Terra e pela Lua. Escreva c) Força do Sol sobre a Lua.
se cada uma das forças seguintes é uma força interna ou Pelo mesmo motivo da questão b, concluímos que essa força é
externa a esse sistema. externa.
a) Força da Terra sobre a Lua.
Esta é uma força entre corpos pertencentes ao sistema. Logo, é
uma força interna.
d) Força da Lua sobre a Terra.
b) Força do Sol sobre a Terra. Pelo mesmo motivo da questão a, concluímos que esta força é

O Sol não pertence ao sistema considerado. Logo, a força que ele interna.

exerce sobre a Terra é uma força externa.

10 Conservação da quantidade de movimento


6 Sobre uma mesa horizontal, há três esferas de aço, A, B e C, 7 Suponha que as esferas A, B e C do exercício anterior estejam
cujas massas são mA 5 2,0 kg, mB 5 0,50 kg e mC 5 2,0 kg. m
Ene-5
ligadas umas às outras por meio de elásticos esticados, que
C 8
Em um dado instante, as esferas possuem as velocidades H-1 exercem forças sobre elas. Considere o sistema constituído
indicadas na figura deste exercício. Nesse instante: pelas esferas e pelos elásticos e suponha que nenhuma outra
força esteja atuando nesse sistema.

1,0 m/s a) As forças exercidas pelos elásticos sobre as esferas são


A
internas ou externas?
3,0 m/s Essas forças são exercidas entre elementos pertencentes ao sis-
C
tema. Logo, são forças internas.
4,0 m/s
B

a) calcule os valores dos módulos das quantidades de


movimento q&a, q&b e q&c de cada esfera. Faça uma cópia
da figura e desenhe nela esses vetores. b) As quantidades de movimento q&a, q&b e q&c estão variando?
Temos: Explique.
qA 5 mAvA 5 2,0 ? 1,0 [ qA 5 2,0 kg ? m/s
As forças internas que atuam nas esferas provocam variações em
qB 5 mBvB 5 0,50 ? 4,0 [ qB 5 2,0 kg ? m/s
qC 5 mCvC 5 2,0 ? 3,0 [ qC 5 6,0 kg ? m/s suas quantidades de movimento, isto é, há variações nas quan-
Percebe-se facilmente que os vetores q&A e q&B estão voltados para tidades de q&a, q&b e q&c.
a esquerda e o vetor q&C aponta para a direita.

c) A quantidade de movimento total Q& do sistema está va-


riando? Explique.
b) quais são o módulo, a direção e o sentido da quantidade Não há forças externas atuando no sistema. Então, apesar de
de movimento do sistema constituído pelas esferas A e B? ocorrerem variações em q&a, q&b e q&c, podemos afirmar que a quan-
Os vetores q&A e q&B estão ambos voltados para a esquerda. Então,
a quantidade de Q&' sistema de bolas A e B terá um módulo: tidade de movimento total Q& permanece constante.
Q' 5 qA 1 qB 5 2,0 1 2,0 [ Q' 5 4,0 kg ? m/s
Evidentemente, Q&' está voltado para a esquerda.

8 (UFTM-MG) Num jogo de sinuca, a bola branca é lançada


m
Ene-5
com velocidade v1 de módulo igual a 2 m/s contra a bola
C 2
H-2 preta, que está em repouso no ponto P, colidindo com ela
m nesse ponto.
Ene-5
C 7
c) determine, em módulo, direção e sentido, a quantidade H-1
de movimento total Q& do sistema constituído pelas três y
esferas.
O vetor Q& será a resultante dos vetores q&C e Q&'. Como q&C aponta
P Tabela
FRENTE B
para a direita e Q&' aponta para a esquerda, temos:
Q 5 qC 2 Q' 5 6,0 2 4,0 [ Q 5 2,0 kg ? m/s
Como qC . Q', concluímos que o vetor Q& aponta para a direita.
Antes da
colisão
FÍSICA

v1

60°
x

Conservação da quantidade de movimento 11


Imediatamente após a colisão, as bolas movem-se perpendicularmente uma à outra, a bola branca com velocidade v'1 de módulo
igual a 3 m/s e a bola preta com velocidade v'2 dirigindo-se para a caçapa, numa direção perpendicular à tabela, conforme indica
a figura.
v'1

Depois da
colisão

P v'2

Considerando que as bolas tenham massas iguais, o módulo de v'2 em m/s, é: c


a) 3.
b) 2.
c) 1.
d) 3.
2
e) 0,5.
Pela conservação da quantidade de movimento, as componentes da quantidade de movimento antes do choque devem ser iguais às compo-
nentes após o choque.
Qantes 5 Qdepois
mv1 5 mv'1 1 mv12
v1 cos 60° 5 v'2
v'2 5 2 ? 0,5
v'2 5 1 m/s

TAREFA PARA CASA: Para praticar: 4 a 7 Para aprimorar: 3 e 4

CONSERVAÇÃO DA QUANTIDADE DE MOVIMENTO


Como vimos, as forças internas não provocam variação na quantidade de movimento total, Q&,
de um sistema. Portanto, qualquer variação em Q& só poderá ser causada por forças externas. Se não
atuarem forças externas em um sistema, ou se a resultante das forças externas atuantes for nula, não
poderá haver variação em Q&, isto é, a quantidade de movimento do sistema permanecerá constante.
Chegamos, assim, às condições para que a quantidade de movimento se conserve.

Se for nula a resultante das forças externas que atuam em um sistema de partículas, a
quantidade de movimento total desse sistema se conservará.

Suponha que você esteja sobre um skate que está em repouso. Qual é a quantidade de movi-
mento deste sistema (você e o skate)? Como o skate está em repouso, a quantidade de movimento
é zero. Agora, suponha que você suba no skate e dê um impulso para colocá-lo em movimento,
conforme a figura 6. O que acontece com o skate? Quando você dá o impulso em um sentido, o
skate se move para o sentido oposto, ou seja, o impulso provocou uma variação na quantidade de
movimento. Esta situação é um exemplo de conservação da quantidade de movimento também
conhecido como conservação do momento linear.

12 Conservação da quantidade de movimento


WILLIAM PERUGINI/SHUTTERSTOCK/GLOW IMAGES
Fig. 6 – O garoto exerce uma força impulsionando o skate com o pé durante um intervalo de tempo.
O deslocamento do skatista é em um sentido, enquanto o skate se move em sentido oposto.

Devemos perceber que as condições para a conservação da quantidade de movimento são


bem mais amplas que as condições para a conservação da energia mecânica. A energia mecânica se
conserva se atuarem sobre ela apenas forças conservativas. Entretanto, a quantidade de movimento
se conservará mesmo que estejam atuando sobre ela forças dissipativas, como o atrito, desde que
sejam forças internas ao sistema.
A seguir, apresentaremos dois exercícios resolvidos que lhe ajudarão a compreender como a
conservação da quantidade de movimento pode ser usada na solução de problemas.

EXERCÍCIOS RESOLVIDOS

4 Um bloco de massa m está em repouso. Num certo instante, RESOLUÇÃO:


m 2m A quantidade de movimento do bloco, imediatamente antes
ele explode em duas partes, A e B, de massas e , res-
3 3 da explosão, é igual à quantidade de movimento imediata-
pectivamente. Sendo vA 5 20 m/s o módulo da velocidade mente depois:
de A, imediatamente após a explosão, qual é o módulo da
Qantes 5 Qdepois
velocidade que B adquire imediatamente após a explosão?
Estando inicialmente em repouso, temos Qantes 5 0.
Portanto: FRENTE B
vA& vB& Qdepois 5 qA 1 qB 5 0 ⇒ mA ? vA 1 mB ? vB 5 0 ⇒
m A B
⇒ mA ? vA 5 2mB ? vB
As partes A e B têm quantidades de movimentos de mesma
FÍSICA

direção, sentidos opostos e módulos iguais:


Repouso m 2m
mA ? vA 5 mB ? vB ⇒ ? 20 5 ? vB ⇒ vB 5 10 m/s
(antes) (depois) 3 3

Conservação da quantidade de movimento 13


5 A situação A da figura abaixo mostra dois blocos, A e B, em um certo tempo, o bloco e a prancha atingem a mesma
repouso, encostados em uma mola comprimida, de massa velocidade final v &2, passando a se mover juntos (situação B
desprezível. Os blocos estão apoiados em uma superfície da figura).
sem atrito e suas massas são mA 5 5,0 kg e mB 5 7,0 kg.
A
Abandonando o sistema, a mola se distende, empurrando
os blocos (situação B da figura). Supondo que o bloco B
adquira uma velocidade vB 5 2,0 m/s, qual será a velocidade
vA adquirida pelo bloco A? v&1

AVITS ESTÚDIO GRÁFICO/


ARQUIVO DA EDITORA
B
B
A
v&2

B
B
A a) Qual é o valor da velocidade v&2?
b) Qual é a quantidade de calor gerado pelo atrito entre o
bloco e a prancha?
RESOLUÇÃO:
RESOLUÇÃO:
Consideremos o sistema constituído pelos dois blocos e
pela mola. A resultante das forças externas que atuam no a) Vamos adotar como sistema o conjunto prancha-bloco.
sistema é nula: os pesos dos blocos e as reações normais A resultante das forças externas (pesos e reação nor-
da superfície se anulam. Logo, a quantidade de movimen- mal) é nula. As forças de atrito entre o bloco e a prancha
to do sistema tem o mesmo valor em qualquer instan- são forças internas e, portanto, não provocam variação
te, embora a quantidade de movimento de cada bloco na quantidade de movimento do sistema. Sendo Q&1 a
varie em virtude da ação das forças internas exercidas quantidade de movimento do sistema no instante inicial
pela mola sobre eles. Designando por Q&1 a quantidade de e Q&2 a quantidade de movimento final, devemos ter:
movimento inicial do sistema (instante em que os blocos Q&2 5 Q&1
foram abandonados) e por Q&2 a quantidade de movimen-
Como a prancha inicialmente estava em repouso, o valor
to final (instante em que os blocos abandonam a mola),
de Q&1se refere apenas ao movimento do bloco, isto é:
devemos ter:
Q1 5 m1v1 5 5,0 ? 6,0 [ Q1 5 30 kg ? m/s
Q&2 5 Q&1 No fim o bloco e a prancha deslocam-se com a mesma
Mas Q&1 5 0, pois os blocos, ao serem abandonados, estavam velocidade v&2. Logo:
em repouso e Q&2 5 q&A 1 q&B, em que q&A e q&B são as quantida- Q2 5 (m1 1 m2)v2 5 (10 1 5,0)v2 [ Q2 5 15v2
des de movimento adquiridas por A e B. Logo:
Como Q1 5 Q2, temos:
q&A 1 q&B 5 0 ou mAv&A 1 mBv&B 5 0 15v2 5 30 [ v2 5 2,0 m/s
mB vB&
∴ vA& 5 2 b) Sendo a energia cinética dada por ε 5 1 mv 2 , e a energia
mA 2
cinética inicial do sistema devida apenas ao movimento
O sinal negativo na expressão anterior nos mostra que v &A tem do bloco, temos:
sentido contrário a v&B. O módulo de v&A será:
εc1 5 1 mv12 5 1 ? 5, 0 ? ( 6, 0)2 ∴ εc1 5 90 J
mB vB 7,00 ? 2 ,0 2 2
vA 5 5 ∴ vA 5 2,8 m/s
mA 5,0 Portanto, a energia cinética final do sistema será:
εc2 5 1 mv 22 5 1 ? (5,0 110
1 10 ) ? ( 2, 0)2 ∴ εc2 5 30 J
6 Uma prancha cuja massa é de 10 kg está em repouso 2 2
sobre uma superfície horizontal, sem atrito. Um bloco, de Houve uma diminuição de 60 J na energia cinética do sis-
5,0 kg de massa, é arremessado horizontalmente sobre a tema. Como a energia total sempre se conserva, concluí-
prancha, com uma velocidade v1 5 6,0 m/s (situação A mos que, na interação entre o bloco e a prancha, 60 J de
da figura a seguir). Por causa do atrito entre o bloco e a energia mecânica foram transformados em 60 J de ener-
prancha, esta é arrastada e entra em movimento. Após gia térmica devido ao atrito.

14 Conservação da quantidade de movimento


PARA CONSTRUIR

9 A figura deste exercício representa duas bolas de bilhar, A d) Sabendo-se q&'A 5 1,0 kg ? m/s, qual é o valor de q&'B?
e B, que se movimentam inicialmente com quantidades É claro que Qf 5 q'A 1 q'B. Então:
de movimento qA 5 2,5 kg ? m/s e qB 5 1,5 kg ? m/s. A 4,0 5 1,0 1 q'B [ q'B 5 3,0 kg ? m/s

bola A atinge a bola B e, após a colisão, as duas passam a


se mover com quantidades de movimento q&'A e q&'B, como
mostra a figura.

q&A q&B
AVITS ESTÚDIO GRÁFICO/
ARQUIVO DA EDITORA

A B

Inicial

e) Supondo que a massa de B seja de 0,50 kg, qual é o valor


q&'A q&'B da velocidade final dessa bola?
A B Sendo v'B a velocidade final de B, temos:
q'B 5 mB v'B ⇒ 3,0 5 0,50 ? v'B [ v'B 5 6,0 m/s
Final

Considerando o sistema constituído pelas duas bolas, res-


ponda:
a) Qual é a quantidade de movimento inicial do sistema?
Observando que q&'A e q&'B têm a mesma direção e o mesmo 10 (Fuvest-SP) Um trabalhador de massa m está em pé, em
sentido, podemos escrever:
Qi 5 qA 1 qB 5 2,5 1 1,5 [ Qi 5 4,0 kg ? m/s m
Ene-5
repouso, sobre uma plataforma de massa M. O conjunto se
C 1
H2
- move, sem atrito, sobre trilhos horizontais e retilíneos, com
m velocidade de módulo constante v. Num certo instante,
Ene-5
C 7
H-1 o trabalhador começa a caminhar sobre a plataforma e
permanece com velocidade de módulo v, em relação a ela, e
com sentido oposto ao do movimento dela em relação aos
trilhos. Nessa situação, o módulo da velocidade da plataforma
em relação aos trilhos é: a
b) As forças que as bolas exercem, uma sobre a outra, (2m 1 M)v
a) .
durante a colisão, são internas ou externas? (m 1 M)
Essas forças exercidas entre corpos do sistema são forças
(2m 1 M)v
internas.
b) .
M
c) (2m 1 M)v .
c) Supondo que a resultante das forças externas seja nula, m
qual é o valor da quantidade de movimento final do d) (M 2 m)v .
sistema? M
Como não há ação de forças externas, a quantidade de movimento
(m 1 M)v
final, Qf, do sistema deve ser igual à quantidade de movimento e) .
inicial, isto é, deve-se ter Qf 5 4,0 kg ? m/s. (M 2 m)
Pela conservação da quantidade de movimento:
FRENTE B
Q antes 5 Q depois
(m 1 M)v 5 mv h 1 Mv'2
(m 1 M)v 5 m(2v 1 v'2) 1 Mv'2
(m 1 M)v 5 2mv 1 (m 1 M)v'2
FÍSICA

v'2 5 (2m 1 M)v


(m 1 M)

TAREFA PARA CASA: Para praticar: 8 a 11 Para aprimorar: 5 e 6

Conservação da quantidade de movimento 15


Estabelecimento do conceito de quantidade de movimento
Observando os objetos que nos rodeiam, podemos perceber que aqueles que estão
em movimento acabam, depois de um certo tempo, perdendo velocidade e chegando ao
repouso. Os filósofos do século XVII preocupavam-se com esse fato, pois ele parecia indicar
que o “movimento total” do Universo estava diminuindo ou, em outras palavras, que o
“Universo estaria morrendo”. Para esses filósofos, essa ideia era inaceitável, pois, sendo uma
obra divina, o Universo deveria ser eterno.
Vários cientistas e filósofos da época passaram a acreditar na possibilidade da existência de
uma grandeza, relacionada com o movimento, que deveria se manter constante enquanto os
corpos interagiam uns com os outros, mesmo que alguns, eventualmente, acabassem por parar.
Na tentativa de encontrar qual seria a grandeza que permaneceria constante, foi inicialmente
levantada a hipótese de que o vetor velocidade v & satisfizesse essa condição. Embora, em alguns
casos, o vetor velocidade total de corpos que interagem realmente permaneça constante, há
situações em que isso não acontece. Por exemplo, na colisão completamente inelástica de dois
corpos de massas diferentes, que se movimentam inicialmente com velocidades de módulos
iguais e de sentidos contrários (figura a seguir), temos:
v&1 v&2

AVITS ESTÚDIO GRÁFICO/


ARQUIVO DA EDITORA
2m m

Antes: ( v&1 1 v&2) 5 0

v&'1 v&'2

2m m

Depois: (v'&1 1 v'&2) Þ 0

Nesta colisão, não há conservação do vetor velocidade.


Antes da colisão: v&1 1 v&2 5 0
Depois da colisão: v&'1 1 v&'2 ± 0
Logo, a velocidade vetorial total não se conservou durante a colisão e podemos concluir
que essa não é a grandeza que permaneceria constante nas interações dos corpos.
O grande filósofo e cientista francês René Descartes, interessando-se pelo
problema, sugeriu que a grandeza procurada deveria ser obtida multiplicando-se a
massa m do corpo pelo módulo v de sua velocidade. Ele acreditava que essa grandeza
permaneceria constante nas interações entre os corpos, denominando-a “quantidade
de movimento” do corpo. Portanto, segundo Descartes, a quantidade de movimento
seria uma grandeza escalar, q, dada por q 5 mv.
Apesar da reconhecida genialidade de Descartes, sua proposta não estava correta,
tendo sido duramente criticada pelo grande matemático alemão Leibniz. Com exemplos
simples, Leibniz apresentou vários tipos de colisões em que a grandeza escalar q 5 mv não
se conservava, ao contrário do que supunha Descartes.
A maneira adequada de medir a “quantidade de movimento” por meio de uma
grandeza cujo valor total se conservasse nas interações dos corpos só veio a ser
encontrada, alguns anos mais tarde, por Isaac Newton, que definiu a “quantidade de
movimento” da maneira apresentada no texto deste capítulo, isto é, q& seria uma grandeza
vetorial, dada pela relação q& 5 mv .&
Realmente, como vimos, o valor total dessa grandeza se conserva em qualquer tipo de
colisão e nas interações entre corpos de um sistema isolado. Em outras palavras, a quantidade
de movimento total do Universo permanece constante no decorrer do tempo. Estava,
portanto, resolvido o problema que tanto preocupou os filósofos do século XVII.

16 Conservação da quantidade de movimento


COLISÕES
Análise de uma colisão
Uma das aplicações mais importantes do conceito de quantidade de movimento é encontrada
no estudo de interações de curta duração, entre partes de um sistema (ou conjunto) de corpos,
como ocorre em uma explosão ou em uma colisão.
Antes: Q1 5 4 kg ? m/s
A
mA

mB
v&1A
v&1B

Depois: Q2 5 4 kg ? m/s
B mA
v2A , v2B
mB
v&2A
v&2B

Fig. 7 – Em uma colisão, a quantidade de movimento total se conserva.

Para entender como essa grandeza está envolvida nesses fenômenos, consideremos a figura 7,
que mostra duas esferas A e B deslocando-se ao longo de uma mesma reta, inicialmente em sentidos
contrários. Após colidirem, as esferas passam a se mover no mesmo sentido. Suponha que as massas
das esferas sejam mA 5 2 kg e mB 5 1 kg, e suas velocidades, antes da colisão, fossem v1A 5 3 m/s e
v1B 5 2 m/s, com os sentidos indicados na figura 7-A.
Em uma situação real, efetuando medidas cuidadosas, uma pessoa encontrou os seguintes
valores para as velocidades das esferas após a colisão: v2A 5 0,5 m/s e v2B 5 3 m/s, com os sentidos
indicados na figura 7-B.
Determinando a quantidade de movimento total, Q&, do conjunto (ou sistema) constituído
pelas duas esferas, antes e depois da colisão, temos:
antes da colisão (note que os vetores q&1A e q&1B têm sentidos contrários):
Q1 5 mA ? v1A 2 mB ? v1B 5 2 ? 3 2 1 ? 2 5 6 2 2
Q1 5 4 kg ? m/s
depois da colisão (note que os vetores q&2A e q&2B têm o mesmo sentido):
Q2 5 mA ? v2A 1 mB ? v2B 5 2 ? 0,5 1 1 ? 3 5 1 1 3
Q2 5 4 kg ? m/s
Logo: Q2 5 Q1
Portanto, as quantidades de movimento do sistema são iguais (em módulo, direção e sentido)
FRENTE B
antes e depois da colisão. Em outras palavras, a quantidade de movimento total, Q&, do sistema
constituído pelos dois corpos que colidiram se conservou durante a colisão.

Colisões diretas e oblíquas


FÍSICA

Quando dois corpos colidem, como no choque entre duas bolas, pode acontecer que a direção
do movimento dos corpos não seja alterada pelo choque, isto é, eles se movimentam sobre uma
mesma reta antes e depois da colisão (fig. 8-A). Quando isso acontece, dizemos que ocorreu uma
colisão direta ou uma colisão central ou, ainda, que ocorreu um choque unidimensional.

Conservação da quantidade de movimento 17


Se, entretanto, os corpos se movimentarem em direções diferentes, antes ou depois da colisão
(fig. 8-B), esta é denominada colisão oblíqua ou choque bidimensional.
A Antes Depois

v1B v2A

v1A v2B
mA mB mA mB

mA
B Antes Depois

v1B v2A

u
v1A
mA mB
v2B
Fig. 8 – Em (A), as esferas realizam uma
colisão direta e, em (B), uma colisão oblíqua. mB

Colisões elásticas e inelásticas


Consideremos a colisão entre duas bolas de bilhar representada na figura 9. Suponha que as
energias cinéticas dos corpos, antes da colisão, fossem εcA 5 8 J e εcB 5 4 J e que, após o choque,
passaram a ser ε'cA 5 5 J e ε'cB 5 7 J. Observe que, antes da colisão, a energia cinética total do sistema era:
εcA 1 εcB 5 8 J 1 4 J 5 12 J
Calculando a energia cinética do sistema após a colisão, verificamos que:
ε’cA 1 ε’cB 5 5 J 1 7 J 5 12 J

AVITS ESTÚDIO GRÁFICO/


ARQUIVO DA EDITORA
εcA 5 8 J A
ε'cA 5 5 J

εcB 5 4 J
ε'cB 5 7 J
Fig. 9 – Em uma colisão elástica, a
energia cinética do sistema se conserva.

Portanto, nessa colisão, a energia cinética total tem o mesmo valor antes e depois do
choque, isto é, a energia cinética do sistema se conservou. Sempre que isso ocorre, dizemos que
a colisão é elástica. De um modo geral, o choque é elástico quando os corpos que colidem não
sofrem deformações permanentes durante a colisão. Duas bolas de bilhar, por exemplo, realizam
colisões que podem ser consideradas elásticas.
Em caso contrário, se os corpos apresentarem deformações permanentes em virtude da colisão,
ou se houver produção de calor durante o choque, verificamos que haverá uma redução no valor
da energia cinética do sistema, pois parte dessa energia cinética terá sido utilizada para produzir as
deformações ou transformada em energia térmica. Sempre que os valores da energia cinética do
sistema, antes e depois da colisão, forem diferentes, dizemos que a colisão é inelástica.
Um caso particular de colisão inelástica ocorre quando os corpos, após o choque, passam a ter
velocidades iguais (fig. 10). Isso ocorre, por exemplo, quando dois automóveis colidem e passam a se
mover colados após o choque. Nesse caso, verifica-se a maior redução possível no valor da energia
cinética do sistema. Por isso, essa colisão é denominada completamente inelástica.

18 Conservação da quantidade de movimento


mA mB

CASA DE TIPOS/ARQUIVO DA EDITORA


vA vB

Antes

Colisão inelástica

vAB
mA 1 mB

Fig. 10 – Colisão inelástica entre dois veículos.


Após o choque, os veículos passam a ter a
Depois mesma velocidade vAB, pois se movem juntos.

Conservação da quantidade de movimento nas colisões


Acabamos de ver que a energia cinética total nem sempre se conserva em uma colisão.
Entretanto, se calcularmos a quantidade de movimento total dos corpos, antes e depois de colidirem,
verificaremos, qualquer que seja a colisão, que esta quantidade de movimento se conserva.
Procuraremos entender por que isso ocorre.
No caso de não existirem forças externas atuando sobre os corpos que colidem, é natural que
isso ocorra, pois já sabemos que a quantidade de movimento de um sistema se conserva se nele
atuarem apenas forças internas. Contudo, mesmo que existam forças externas, como a duração
do choque é sempre muito curta, o impulso exercido por essas forças externas será também muito
pequeno (em geral, os valores das forças externas não são muito grandes) e, consequentemente, a
variação da quantidade de movimento que elas provocam pode ser desprezada.
Observe que as forças impulsivas que ocorrem nas colisões (ou explosões), por serem muito
grandes, podem provocar variações apreciáveis nas quantidades de movimento de cada um dos corpos
que colidem, mas, como são forças internas, elas não influenciarão na quantidade de movimento total.
Assim, podemos concluir que as quantidades de movimento de um sistema, imediatamente antes e
depois de qualquer colisão, podem ser consideradas iguais. Destacamos, portanto, que:

Considerando um sistema de corpos que colidem, a quantidade de movimento


total imediatamente antes da colisão é igual à quantidade de movimento total do sistema
imediatamente após a colisão.

O número de problemas que podem ser resolvidos usando-se esta conclusão é muito grande.
Os exercícios resolvidos a seguir ilustram como a conservação da quantidade de movimento pode
ser usada na solução de problemas de colisões ou explosões. FRENTE B

A B
FÍSICA
AVITS ESTÚDIO GRÁFICO/

No dispositivo representado na figura, todas as esferas (de aço) têm


ARQUIVO DA EDITORA

a mesma massa. Levantando a esfera de uma das extremidades e


deixando-a colidir com as demais, como em (A), observamos que
ela entra em repouso e apenas a bola da outra extremidade adquire
uma velocidade igual à da esfera incidente (B). A quantidade de
movimento se conserva.

Conservação da quantidade de movimento 19


EXERCÍCIOS RESOLVIDOS

7 Em uma mesa de sinuca, a bola branca, de massa m, movendo-se com velocidade v 5 2,0 m/s, atinge a bola amarela
(também de massa m), que estava em repouso. Supondo que o choque seja central e elástico, determine as velocidades das
duas bolas após a colisão.

RESOLUÇÃO:
Sejam v1& e v2& as velocidades das bolas branca e amarela após a colisão. A quantidade de movimento do sistema (das duas bolas),
antes do choque, era mv,& pois apenas a bola branca estava em movimento. Como em qualquer colisão há conservação da quan-
tidade de movimento total, podemos escrever: mv & 5 mv1& 1 mv2& .
Já que o choque é central, os vetores v,& v1& e v2& têm a mesma direção e, portanto, a relação anterior poderá ser escrita escalarmente,
isto é: mv 5 mv1 1 mv2 ⇒ v 5 v1 1 v2 [ v1 1 v2 5 2,0.
Além disso, tratando-se de uma colisão elástica, a energia cinética do sistema se conserva. Logo:

1 mv 2 5 1 mv 2 1 1 mv 2 ⇒
2 2 1 2 2
⇒ v 2 5 v12 1 v 22 ∴ v12 1 v 22 5 4 ,0

Obtivemos, assim, duas equações, relacionando as incógnitas v1 e v2: v1 1 v2 5 2,0 e v21 1 v22 5 4,0.
Da primeira equação, v1 5 2,0 2 v2; substituindo-a na segunda: (2,0 2 v2)2 1 v22 5 4,0.
Resolvendo essa equação, obtemos v2 5 2,0 m/s e, como v1 5 2,0 2 v2, concluímos que v1 5 0.
Assim, percebemos que, por causa da colisão, a bola branca entra em repouso e a bola amarela adquire uma velocidade igual à que
a bola branca possuía antes do choque. É possível que você já tenha visto esse fato ocorrer em um jogo de bilhar.

8 Suponha que uma pedra, em repouso, seja fragmentada em três pedaços por uma explosão. Um dos pedaços, de massa
m1 5 1,0 kg, parte com uma velocidade v1 5 12 m/s. Um segundo pedaço, de massa m2 5 2,0 kg, sai com velocidade v2 5 8,0 m/s,
em uma direção perpendicular a v&1.
v2&
a) Desenhe um diagrama que mostre a direção do movimento do terceiro pedaço, imediata-
mente após a explosão.
b) Se a massa do terceiro pedaço era m3 5 0,50 kg, qual é a velocidade, v &3, desse pedaço, imedia-
tamente após a explosão?

RESOLUÇÃO:
a) A quantidade de movimento do sistema (a pedra), antes da explosão, era nula. Como a explo- v1&
são dura um intervalo de tempo muito curto, as forças externas não alterarão sensivelmente o
vetor Q& e, assim, a quantidade de movimento do sistema deverá ser ainda nula imediatamen-
te após a explosão. As quantidades de movimento adquiridas pelo primeiro e pelo segundo
pedaço valem, respectivamente:
q1 5 m1v1 5 1,0 ? 12 [ q1 5 12 kg ? m/s v3&
q2 5 m2v2 5 2,0 ? 8,0 [ q2 5 16 kg ? m/s
Na figura ao lado, foram traçados, em escala, os vetores q&1 e q&2. Para que a quantidade de movi-
mento total Q& seja nula, a quantidade de movimento q&3, do terceiro pedaço, deverá ser igual e
contrária à resultante de q&1 e q&2. Portanto, o terceiro pedaço se movimentará na direção do vetor
q &2
q&3 mostrado na figura.
b) Como vimos, o vetor q&3 tem o mesmo módulo da soma vetorial q&1 e q&2 Pela figura acima:
q3 5 q12 1 q22 5 122 1 162 ∴ q3 5 20 kg ? m/s
q &1
Mas:
q3 5 m3v3 ou 20 5 0,50 ? v3 [ v3 5 40 m/s q &3

Portanto, o terceiro pedaço parte, logo após a explosão, com uma velocidade de 40 m/s na
direção e no sentido do vetor q&3 na figura ao lado.

20 Conservação da quantidade de movimento


PARA CONSTRUIR

11 Observe a figura deste exercício. A situação (1) mostra duas 12 Duas locomotivas, A e B, movem-se no mesmo sentido ao
bolas de aço imediatamente antes e imediatamente após m
Ene-5
longo de um trilho reto e horizontal, estando A à frente de B.
C 1
colidirem entre si. A situação (2) representa uma bala disparada H-2 Sabe-se que mA 5 3,0 ? 105 kg, vA 5 8,0 m/s, mB 5 5,0 ? 105 kg
contra um bloco de madeira imediatamente antes da colisão e m e vB 5 16 m/s. A locomotiva B choca-se com A, passando am-
Ene-5
movendo-se com o bloco imediatamente após penetrar nele. C 7
H-1 bas a se deslocar juntas após a colisão.
Para cada uma das situações descritas, responda: a) Como você classifica essa colisão?
εCA 5 12 J εCB 5 15 J ε'CA 5 18 J ε'CB 5 9 J
Como as duas locomotivas movem-se juntas após o choque,
A B A B
dizemos que se trata de uma colisão completamente inelástica.

1 Inicial Final Além disso, elas se movimentam ao longo da mesma reta


antes e depois do choque. Então, a colisão é central (ou direta).

2 Inicial Final

a) Durante a colisão, a energia cinética do sistema se


conservou?
Para a situação (1), temos:
εcA 1 εcB 5 12 J 1 15 J 5 27 J
ε'cA 1 ε'cB 5 18 J 1 9 J 5 27 J
Portanto, houve conservação da εc do sistema nesta colisão. b) Qual é a quantidade de movimento do sistema constituído
Por outro lado, quando dois corpos movem-se juntos após a pelas duas locomotivas imediatamente antes da colisão?
colisão, há a maior dissipação possível de εc. Portanto, na situação Como v&A e v&B têm a mesma direção e o mesmo sentido, teremos:
(2), não há conservação da εc do sistema. Q1 5 mAvA 1 mBvB 5 3,0 ? 105 ? 8,0 1 5,0 ? 105 ? 16
[ Q1 5 104 ? 105 kg ? m/s 5 1,04 ? 107 kg ? m/s

b) A colisão é elástica, inelástica ou perfeitamente inelástica?


Na situação (1), temos uma colisão elástica (há conservação c) Qual deve ser o valor da quantidade de movimento do
sistema imediatamente após a colisão?
da εc do sistema) e, na situação (2), como dissemos, temos uma A quantidade de movimento total deve se conservar durante a
colisão perfeitamente inelástica. colisão. Então, a quantidade de movimento do sistema, logo após
o choque das locomotivas, deve ser:
Q2 5 1,04 ? 107 kg ? m/s

d) Considerando a resposta da questão c, determine a


c) A quantidade de movimento do sistema se conservou
velocidade com que as locomotivas se movem logo após
durante a colisão? Explique.
a colisão.
Em qualquer tipo de colisão, podemos sempre considerar que a Sendo v& a velocidade com que as locomotivas se movimentam
quantidade de movimento do sistema se conserva. Então,
após o choque, temos: FRENTE B
Q2 5 (mA 1 mB)v
tanto na situação (1) como na situação (2), podemos dizer que Portanto:
1,04 ? 107 5 (8,0 ? 105) ? v [ v 5 13 m/s
Q& permanece constante.
FÍSICA

Conservação da quantidade de movimento 21


13 (Fuvest-SP) Compare as colisões de uma bola de vôlei e de Note e adote: A massa da pessoa é muito maior que a massa
m uma bola de golfe com o tórax de uma pessoa parada e em das bolas. As colisões são frontais. O tempo de interação da
Ene-5
C 1
H-2 pé. A bola de vôlei, com massa de 270 g, tem velocidade de bola de vôlei com o tórax da pessoa é o dobro do tempo de
30 m/s quando atinge a pessoa, e a de golfe, com 45 g, tem interação da bola de golfe. A área média de contato da bola
m
Ene-5 de vôlei com o tórax é 10 vezes maior que a área média de
C 7 velocidade de 60 m/s ao atingir a mesma pessoa nas mesmas
H-1
condições. Considere ambas as colisões totalmente inelásti- contato da bola de golfe.
Pelo teorema do impulso, temos:
cas. É correto apenas o que se afirma em: e I 5 ∆q
a) Antes das colisões, a quantidade de movimento da bola Fm ? ∆t 5 m∆v
m | ∆v |
de golfe é maior que a da bola de vôlei. Fm 5
∆t
b) Antes das colisões, a energia cinética da bola de golfe é F
A pressão é dada por pm 5 m .
maior que a da bola de vôlei. A
m | ∆v |
c) Após as colisões, a velocidade da bola de golfe é maior Relacionando as duas equações acima, temos: pm 5 .
∆t ? A
que a da bola de vôlei. Logo, a razão entre a bola de golfe e a bola de vôlei é:
d) Durante as colisões, a força média exercida pela bola de mG | ∆v G | 45 | 0 2 60 |
golfe sobre o tórax da pessoa é maior que a exercida pela p mG ∆t G ? A G ∆t G ? A G
5 5 5 20 5 6,7
bola de vôlei. p mV m V | ∆v V | 270 | 0 2 30 | 3
∆t V ? A V 2∆t G ? 10A G
e) Durante as colisões, a pressão média exercida pela bola de
golfe sobre o tórax da pessoa é maior que a exercida pela p mG 5 6,7p mV
bola de vôlei. p mG . p mV

TAREFA PARA CASA: Para praticar: 12 a 15 Para aprimorar: 7 a 12

Veja, no Guia do Professor, as respostas da “Tarefa para casa”. As resoluções encontram-se no portal, em Resoluções e Gabaritos.

TAREFA PARA CASA


As resoluções dos exercícios encontram-se no portal, em Resoluções e Gabaritos.

d) diminuirá gradativamente a sua velocidade, pois o impul-


PARAPARA PRATICAR
PRATICAR so resultante será contrário ao seu movimento.
e) aumentará gradativamente a sua velocidade, pois não ha-
1 (Enem) Para entender os movimentos dos corpos, Galileu verá nenhum impulso contrário ao seu movimento.
m
Ene-5
discutiu o movimento de uma esfera de metal em dois
C 7
H-1 planos inclinados sem atritos e com a possibilidade de 2 (Fatec-SP) O Brasil pretende construir um submarino nuclear
m se alterarem os ângulos de inclinação, conforme mostra que terá massa aproximada de 6 000 toneladas e poderá
Ene-6
C 0
H-2 a figura. Na descrição do experimento, quando a esfera descer até uma profundidade de 350 metros e desenvolver
de metal é abandonada para descer um plano inclinado uma velocidade máxima aproximada de 12 m/s.
de um determinado nível, ela sempre atinge, no plano Suponha as duas situações a seguir:
ascendente, no máximo, um nível igual àquele em que foi I. que o submarino descrito esteja completamente estático,
abandonada. totalmente submerso e próximo à lâmina d’água, em uma
determinada região do oceano que possui campo gravi-
Nível de abandono
da esfera
tacional constante;
II. que o submarino descrito esteja navegando à velocida-
de máxima de forma constante, totalmente submerso e
Ângulo do plano de Ângulo do plano de
subida descida
próximo à lâmina d’água, em uma determinada região do
oceano que possui campo gravitacional constante.
Galileu e o plano inclinado. Disponível em: ,www.fisica.ufpb.br>.
Acesso em: 2 mar. 2015. Adaptado. Desprezando a coluna d’água acima do submarino, pode-
Se o ângulo de inclinação do plano de subida for reduzido a mos afirmar que o empuxo produzido na situação (I) e a
zero, a esfera: quantidade de movimento gerada na situação (II), respectiva
a) manterá sua velocidade constante, pois o impulso resul- e aproximadamente, são:
tante sobre ela será nulo. a) 6,0 ? 104 N e 7,2 ? 104 kg ? m/s.
b) manterá sua velocidade constante, pois o impulso da des- b) 6,0 ? 104 N e 7,2 ? 105 kg ? m/s.
cida continuará a empurrá-la. c) 6,0 ? 107 N e 7,2 ? 106 kg ? m/s.
c) diminuirá gradativamente a sua velocidade, pois não ha- d) 6,0 ? 107 N e 7,2 ? 107 kg ? m/s.
verá mais impulso para empurrá-la. e) 6,0 ? 107 N e 7,2 ? 108 kg ? m/s.

22 Conservação da quantidade de movimento


3 (Unifesp) Uma empresa de demolição utiliza um guindaste, 5 Considere um sistema constituído por um automóvel,
m
Ene-5
extremamente massivo, que se mantém em repouso e de massa m1 5 8,0 ? 102 kg, e um caminhão, de massa
C 7
H-1 em equilíbrio estável no solo durante todo o processo. m2 5 2,0 ? 103 kg. Determine o módulo da quantidade
m Ao braço superior fixo da treliça do guindaste, ponto O, de movimento total, Q&, do sistema, em cada um dos
Ene-5
C 8
H-1 prende -se um cabo, de massa desprezível e inextensível, seguintes casos.
de 10 m de comprimento. A outra extremidade do cabo é
a) O caminhão está em repouso e o carro se desloca com
presa a uma bola de 300 kg que parte do repouso, com o
uma velocidade de 10 m/s.
cabo esticado, do ponto A.
b) O caminhão e o carro se deslocam, na mesma direção e
no mesmo sentido, a 20 m/s.
c) O caminhão e o carro se deslocam a 20 m/s na mesma dire-
O ção, mas em sentidos contrários.
u 10 m

6 Um sistema é constituído de três partículas, A, B e C, de


ARQUIVO DA EDITORA

A 5,2 m m massas mA 5 2 kg, mB 5 2 kg e mC 5 0,5 kg. A figura a


B Ene-5
C 7
CASA DE TIPOS/

H-1 seguir mostra as posições e as velocidades das partículas do


sistema em um instante t1 e no instante t2, posterior a t1.

1 m/s
Sabe -se que a trajetória da bola, contida em um plano A
2 m/s
vertical, do ponto A até o ponto B, é um arco de circun- C B
B
ferência com centro no ponto O; que o módulo da veloci- 0,5 m/s
dade da bola no ponto B, imediatamente antes de atingir C

a estrutura do prédio, é de 2 m/s; que o choque frontal da 4 m/s A


4 m/s
bola com o prédio dura 0,02 s; e que depois desse interva-
1,5 m/s
lo de tempo a bola para instantaneamente. Desprezando t1 t2
a resistência do ar e adotando g 5 10 m/s2, calcule, em
newtons:
a) o módulo da força resultante média que atua na bola no
a) Determine o módulo, a direção e o sentido da quantidade
intervalo de tempo de duração do choque.
de movimento do sistema nos instantes t1 e t2.
b) o módulo da força de tração no cabo no instante em que
a bola é abandonada do repouso no ponto A. b) Baseado em sua resposta à questão (a), que conclusão
você tira sobre a resultante das forças externas que atuam
4 (Fatec-SP) Quatro pessoas estão no interior de um veículo no sistema?
m
Ene-6
em repouso. Em um dado instante, o motorista começa a c) Você acha que houve interação (forças internas) entre as
C 0
H2
- aplicar uma força, com as mãos, no para-brisa dianteiro. partículas? Explique.
Verificando que os outros estavam intrigados, comentou
que a aplicação de uma força irá provocar movimento no 7 Um astronauta, tendo em suas mãos um pequeno objeto,
sistema, constituído de carro e passageiros. Cada um dos está em repouso numa região do espaço onde não existe
passageiros forneceu uma justificativa. nenhuma atração gravitacional sobre ele. Nessa situação, ele
I. Sempre que aplicarmos uma força interna no sistema arremessa o objeto, aplicando-lhe um impulso de 12 N ? s.
(carro 1 ocupantes), este irá adquirir movimento. Considere o sistema astronauta 1 objeto e assinale, entre as
II. A força interna irá provocar a variação na quantidade de afirmativas seguintes, aquela que está ERRADA.
movimento do sistema (carro 1 ocupantes). a) O astronauta recebe, do objeto, um impulso de módulo
III. A aplicação de uma força externa provocará o movi- igual a 12 N ? s.
mento do sistema (carro 1 ocupantes). FRENTE B
b) O objeto passa a se deslocar com uma quantidade de mo-
De acordo com as justificativas, está correto o que se afir- vimento de 12 kg ? m/s.
ma apenas em: c) O módulo da quantidade de movimento adquirida pelo
a) I.
FÍSICA

astronauta é menor do que 12 kg ? m/s.


b) II. d) A quantidade de movimento do sistema, antes de o obje-
c) III. to ser arremessado, era nula.
d) I e II. e) A quantidade de movimento do sistema, após o objeto
e) I e III. ser arremessado, é nula.

Conservação da quantidade de movimento 23


8 (UFPE) O martelo de ferro de 1,5 tonelada, de um bate- 10 (UPE) “Curiosity pousa com sucesso em Marte”. Essa foi a
m
Ene-5
-estaca, cai em queda livre de uma altura de 5,0 m, a partir do m
Ene-5
manchete em vários meios de comunicação na madrugada
C 2 C 7
H-2 repouso, sobre uma estaca de cimento. H-1 do dia 6 de agosto de 2012. O robô da Nasa chamado
m Curiosity foi destinado a estudar propriedades do planeta
Ene-6
C 4
H-2 Marte. Após uma viagem de aproximadamente 9 meses,
m o Curiosity chegou a Marte. Ao entrar na atmosfera do
Ene-6
C 5
H-2
planeta, o robô continuava ligado a pequenos foguetes
que foram usados para desacelerá-lo. Segundos antes
da chegada ao solo, os foguetes foram desconectados e
se afastaram para bem longe. A figura ilustra o sistema
Curiosity 1 foguetes.
CASA DE TIPOS/ARQUIVO DA EDITORA

NASA
Estaca

t (s)

Plataforma

O martelo não rebate após a colisão, isto é, permanece


em contato com a estaca. A força exercida pela estaca so-
bre o martelo varia com o tempo de acordo com o gráfico
a seguir.

Fmáx

Estaca

0 t (s)
0,1 0,2 0,3

Calcule o valor da força máxima Fmáx, em unidades de 103 N.


Despreze todas as perdas de energia existentes entre o mar- A massa dos foguetes varia continuamente, enquanto eles
Plataforma
telo e a guia, bem como com as demais engrenagens. queimam combustível e produzem a exaustão dos gases. A
propulsão dos foguetes que fizeram desacelerar o Curiosity é
9 (UFCG-PB) Num laboratório de Física Nuclear, observa-se que um exemplo notável da:
um núcleo, inicialmente em repouso, desintegra-se emitindo a) Lei da Inércia.
um elétron e um neutrino. A quantidade de movimento do b) Lei de Kepler.
elétron é 1,2 ? 10−22 kg ? m/s, na direção crescente do eixo x, e c) Conservação da Energia.
a do neutrino é de 6,4 ? 10−23 kg ? m/s na direção decrescente d) Conservação da Quantidade de Movimento.
do eixo y. e) Lei da Gravitação Universal.

a) Determine a direção da quantidade de movimento do 11 Uma bala, de 10 g de massa, é atirada por uma espingarda
núcleo residual. de massa igual a 4,0 kg, abandonando o seu cano com uma
b) Considerando a massa do núcleo residual igual a velocidade horizontal de 400 m/s. Determine o módulo da
5,8 ? 10−26 kg, calcule o módulo de sua velocidade. velocidade de recuo da espingarda.

24 Conservação da quantidade de movimento


12 (Unicamp-SP) Muitos carros possuem um sistema de segu- c)
rança para os passageiros chamado airbag. Este sistema con-
siste em uma bolsa de plástico que é rapidamente inflada
quando o carro sofre uma desaceleração brusca, interpondo-
-se entre o passageiro e o painel do veículo. Em uma colisão,
a função do airbag é:
a) aumentar o intervalo de tempo de colisão entre o pas-
sageiro e o carro, reduzindo assim a força recebida pelo d)
passageiro.
b) aumentar a variação de momento linear do passageiro
durante a colisão, reduzindo assim a força recebida pelo
passageiro.
c) diminuir o intervalo de tempo de colisão entre o pas-
sageiro e o carro, reduzindo assim a força recebida pelo
passageiro.
d) diminuir o impulso recebido pelo passageiro devido e)
ao choque, reduzindo assim a força recebida pelo
passageiro.

13 (Enem) O pêndulo de Newton pode ser constituído por cinco


m
Ene-5
pêndulos idênticos suspensos em um mesmo suporte. Em
C 7
H-1 um dado instante, as esferas de três pêndulos são deslocadas
m para a esquerda e liberadas, deslocando-se para a direita e
Ene-6
C 0
H-2 colidindo elasticamente com as outras duas esferas, que 14 (Fuvest-SP) Dois caixotes, A e B, de mesma altura e mesma
inicialmente estavam paradas. m massa, podem movimentar-se sobre uma superfície plana
Ene-5
C 1
H2
- sem atrito. Estando inicialmente A parado, próximo a uma
parede, o caixote B aproxima-se perpendicularmente da
parede, com velocidade v0, provocando uma sucessão de
colisões elásticas no plano da figura.

Parede

v0
A B

O movimento dos pêndulos após a primeira colisão está re-


presentado em: Após todas as colisões, é possível afirmar que os módulos das
a) velocidades dos dois blocos serão aproximadamente:
a) vA 5 v0 e vB 5 0.
v
b) vA 5 0 e vB 5 2v0.
2
c) vA 5 0 e vB 5 2v0.
v v
d) vA 5 0 vB 5 0 .
2 2
e) vA 5 0 e vB 5 v0.
FRENTE B

b) 15 (Uerj) Admita uma colisão frontal totalmente inelástica entre


um objeto que se move com velocidade inicial v0 e outro
objeto inicialmente em repouso, ambos com mesma massa.
FÍSICA

Nessa situação, a velocidade com a qual os dois objetos se


movem após a colisão equivale a:
a) v 0 . b) v 0 . c) 2v0. d) 4v0.
2 4

Conservação da quantidade de movimento 25


A bola 2, de massa m2 5 1 kg, move -se com ângulo de 60°
PARA
PARA APRIMORAR
PRATICAR em relação ao eixo x, com velocidade v2 5 2 m/s. Sabe -se
que o coeficiente de atrito cinético entre as bolas e o piso
1 (UFPE) Uma partícula de massa 0,2 kg move-se ao longo do rugoso é 0,10 sec2 β e a aceleração gravitacional é 10 m/s2.
m
Ene-5
eixo x. No instante t 5 0, a sua velocidade tem módulo 10 m/s Ao colidirem, as bolas permanecem unidas após o choque
C 2
H-2 ao longo do sentido positivo do eixo. A figura a seguir ilustra e movimentam-se em um outro piso rugoso, conforme
m o impulso da força resultante na direção x agindo sobre a mostra a figura. A distância percorrida, em metros, pelo
Ene-6
C 4
H2
- partícula. Qual é o módulo da quantidade de movimento da conjunto bola 1 e bola 2 até parar é igual a:
m partícula (em kg ? m/s) no instante t 5 15 s?
Ene-6
C 5
H-2
I (kg ? m/s) Piso rugoso
m1 1 m2
Piso liso
20 112

m1 v1 5 1 m/s b
1
x
0 5 10 15 60°
t (s)

220

v2 5 2 m/s
m2
2
2 (UFTM-MG) Em algumas circunstâncias, nos deparamos com
situações de perigo e, para esses momentos, são necessários a) 0,2.
equipamentos de segurança a fim de evitar maiores danos. b) 0,5.
Assinale a alternativa que justifica corretamente o uso de c) 0,7.
determinados dispositivos de segurança. d) 0,9.
a) O cinto de segurança e o airbag, utilizados nos auto- e) 1,2.
móveis, servem para amortecer o impacto do moto-
rista em uma colisão e, consequentemente, reduzir a 4 Um menino, cuja massa é de 40 kg, está em um carro que
variação do módulo da quantidade de movimento do se desloca em trilhos horizontais com uma velocidade de
motorista na colisão. 3,0 m/s. A massa do carro é de 100 kg.
b) Um automóvel, ao fazer uma curva com velocidade de a) Suponha que o menino saísse do carro sem exercer nele
módulo constante, varia o módulo da quantidade de qualquer impulso. Qual é a velocidade horizontal com
movimento do motorista, uma vez que a resultante das que o menino chegaria ao solo e com qual velocidade o
forças nele aplicadas é nula devido ao uso do cinto de carro continuaria a se mover?
segurança. b) Se o menino saltar de modo que caia verticalmente, com
c) Em uma atividade circense, o trapezista ao cair do trapé- qual velocidade o carro passará a se mover?
zio é amortecido por uma rede de proteção, responsável
pela anulação da quantidade de movimento devido ao
5 (AFA-SP) Considere duas rampas, A e B, respectivamente de
m
Ene-6
massas 1 kg e 2 kg, em forma de quadrantes de circunferência
impulso que ela lhe aplica, o que não ocorreria se ele caís- C 5
H-2 de raios iguais a 10 m, apoiadas em um plano horizontal e sem
se diretamente no solo.
m atrito. Duas esferas, 1 e 2, se encontram, respectivamente, no
d) O impulso exercido por uma rede de proteção sobre o Ene-5
C 7
H-1 topo das rampas A e B e são abandonadas, do repouso, em
trapezista é igual àquele exercido pelo solo, caso não haja
um dado instante, conforme figura abaixo.
a rede; porém, o tempo de interação entre o trapezista e
a rede é maior, o que faz com que diminua a força média 1 2
exercida sobre o trapezista pela rede, em relação ao solo.
e) Ao cair sobre a rede de proteção, o trapezista recebe da
rede uma força maior do que aquela recebida se caísse no A B
solo, oferecendo a ele maior segurança e diminuindo o
risco de acidente.
Quando as esferas perdem contato com as rampas, elas se
3 (IME-RJ) Duas bolas, 1 e 2, movem-se em um piso movimentam conforme os gráficos de suas posições x, em
m
Ene-5
perfeitamente liso. A bola 1, de massa m1 5 2 kg, move-se no metros, em função do tempo t, em segundos, a seguir repre-
C 7
H-1 sentido da esquerda para direita com velocidade v1 5 1 m/s. sentados.

26 Conservação da quantidade de movimento


x (m) e um corte de 8 cm no supercílio esquerdo do piloto. O piloto
brasileiro ficou inconsciente e seu carro colidiu com a proteção
10
de pneus. A mola que atingiu o piloto era de aço, media 12 cm
5 de diâmetro e tinha, aproximadamente, 800 g. Considerando
que a velocidade do carro de Felipe era de 270 km/h no
0 instante em que ele foi atingido pela mola e desprezando
3 t (s)
2
a velocidade da mola e a resistência do ar, some o valor das
afirmações que forem corretas.
m1
Desprezando qualquer tipo de atrito, a razão das massas (01) A quantidade de movimento (momento linear) trans-
m2 ferida do piloto para a mola foi de, aproximadamente,
m1 e m2 das esferas 1 e 2, respectivamente, é:
75 kg ? m ? s21.
a) 1 . (02) Pode-se dizer que esse tipo de colisão é uma colisão
2 perfeitamente inelástica.
b) 1. (04) Tomando-se o referencial do piloto Felipe Massa, pode-
c) 2. -se dizer que a velocidade da mola era de 2270 km/h.
d) 3 . (08) Considerando que o intervalo de tempo do impacto (a
2 duração do impacto) foi de 0,5 s, a aceleração média da
6 Um foguete, na plataforma de lançamento, possui uma massa mola foi de 150 m/s2.
3
m total (incluindo o combustível) de 4,0 ? 10 kg. Processando- (16) Considerando que, após o fim da colisão, a velocidade
Ene-6
C 0
H-2 -se a combustão, o foguete expele rapidamente 800 kg de da mola em relação ao piloto é nula, e tomando o re-
gás, com uma velocidade de 2,0 ? 103 m/s. Lembrando-se da ferencial do piloto Felipe Massa, pode-se afirmar que a
conservação da quantidade de movimento de um sistema, função horária da posição da mola, após o fim da coli-
determine a velocidade adquirida pelo foguete após ejetar são, foi de segundo grau.
essa massa de gás.
10 (IFSP) Existe um brinquedo de criança que é constituído de
7 (ITA-SP) 100 cápsulas com água, cada uma de massa m 5 1,0 g, m um pêndulo de três bolinhas de mesma massa e compri-
Ene-5
C 7
são disparadas à velocidade de 10,0 m/s perpendicularmente H-1 mentos iguais. A brincadeira consiste em abandonar uma
a uma placa vertical com a qual colidem inelasticamente. bolinha X de uma altura H, acima das outras duas Y e W, que
Sendo as cápsulas enfileiradas com espaçamento de 1,0 cm, estão em repouso (figura 1). Quando a bolinha X colide com
determine a força média exercida pelas cápsulas sobre a placa. as duas, todas ficam grudadas e o conjunto atinge uma altura
h acima da posição inicial de Y e W (figura 2).
8 (Fuvest-SP) Uma das hipóteses para explicar a extinção dos
dinossauros, ocorrida há cerca de 60 milhões de anos, foi a
colisão de um grande meteoro com a Terra. Estimativas indi-
cam que o meteoro tinha massa igual a 1016 kg e velocidade
de 30 km/s imediatamente antes da colisão. Supondo que X

esse meteoro estivesse se aproximando da Terra, numa dire-


ção radial em relação à órbita desse planeta em torno do Sol, H h
para uma colisão frontal, determine: Y W
Dados: A órbita da Terra é circular. Massa da Terra: 6 ? 1024 kg. Fig. 1 Fig. 2
1 megaton 5 4 ? 1015 J é a energia liberada pela explosão de
um milhão de toneladas de trinitrotolueno.
a) a quantidade de movimento qi do meteoro imediatamen-
te antes da colisão;
b) a energia cinética εc do meteoro imediatamenteXantes da
colisão;
c) a componente radial da velocidade da Terra,
H vr, pouco de- h FRENTE B
pois da colisão;
d) a energia εd , em megatons, dissipada na colisão. Y W
Fig. 1 Fig. 2
9 (UEM-PR) Durante o treino classificatório para o Grande Prêmio
FÍSICA

m
Ene-5
da Hungria de Fórmula 1, em 2009, o piloto brasileiro Felipe Se desconsiderarmos qualquer tipo de atrito, o valor de h em
C 8
H-1 Massa foi atingido na cabeça por uma mola que se soltou função de H será de:
do carro que estava logo à sua frente. A colisão com a mola
a) H . b) H . c) H . d) H .
causou fratura craniana, uma vez que a mola ficou ali alojada, 2 3 6 9

Conservação da quantidade de movimento 27


11 (Fuvest-SP) Um fóton, com quantidade de movimento na d) y

direção e no sentido do eixo x, colide com um elétron em


repouso. Depois da colisão, o elétron passa a se mover com
quantidade de movimento q&e no plano xy, como ilustra a
figura abaixo.

y x

q&f

e) y

x q&f
x

12 Resolvendo este problema, você estará aprendendo um


q&e m processo muito simples que pode ser usado na medida da
Ene-5
C 2
H-2 velocidade de uma bala de revólver, por exemplo.
Dos vetores q&f abaixo, o único que poderia representar a dire- m
Ene-5
C 7
ção e o sentido da quantidade de movimento do fóton após H-1

a colisão é:
Dado: o princípio da conservação da quantidade de mo-
vimento é válido também para a interação entre fótons e
elétrons.
a) y

q&f

x h
v&
b) y

m
M

Considere uma bala de massa m, disparada com uma velo-


q&f cidade v&, cujo valor desejamos medir. Fazendo a bala incidir
x contra um bloco de madeira, de massa M, suspenso por um
c) y fio, a bala se engasta no bloco e o conjunto sobe até uma
altura h. Suponha que, em uma experiência, na qual m 5 8 g
e M 5 2,0 kg, tenha-se observado que h 5 20 cm.
a) Sendo vf a velocidade do conjunto bala 1 bloco logo
após a colisão, expresse v em função de vf.
b) Lembrando-se de que a energia cinética com que o conjun-
x to parte após a colisão se transforma em energia potencial,
calcule o valor de vf (considere g 5 10 m/s2).
q&f c) Determine o valor da velocidade v com que a bala foi dis-
parada.

28 Conservação da quantidade de movimento


Professor, oriente os alunos a realizar a experiência com a supervisão de um adulto ou responsável. O objetivo deste experimento é mostrar que, num
sistema em que inicialmente não existe movimento nenhum, quando duas partes diferentes desse sistema começam a se movimentar, existe uma com-
EXPERIMENTANDO pensação: os movimentos ocorrem na mesma direção, porém em sentidos opostos.

Canhão de borrachinha
Ideia do experimento
O experimento consiste em construir um sistema muito similar a um canhão real. Um elástico (em verde) é disposto sobre a base de
madeira como se fosse uma atiradeira que está prestes a impulsionar o projétil (veja a figura a seguir). A linha de costura (em vermelho)
e o palito de fósforo servem para disparar o “tiro” com a menor interferência possível.
Depois de armado o sistema, dispara-se o “tiro” simplesmente queimando a linha que mantém a borrachinha esticada. O que se
observa é que, enquanto o projétil é lançado num sentido, o resto do sistema se move noutro sentido, ou seja, recua.
A ideia aqui é a de explorar a compensação de quantidades de movimentos que ocorre de maneira bem visível neste experimento. O
projétil, mais leve, se desloca com velocidade maior; o resto do sistema, mais pesado, se desloca noutro sentido com velocidade menor.
Material
Uma tábua de madeira leve, de 15 × 10 cm (pode ser, por exemplo, obtida de uma caixa de uva, da parte da madeira mais fina que a
compõe)
Três parafusos ou pregos pequenos
Um elástico
Linha de costura
Fósforos
Projétil (pode ser qualquer coisa passível de ser atirada pela borracha: um apontador de lápis, uma borracha de apagar lápis, dessas que
têm uma capa plástica de proteção – só a borracha ofereceria muito atrito –, etc.)
Lápis (a quantidade deve ser tal que permita à base de madeira se deslocar por toda a distância que esta conseguir se mover após o
tiro. Algo como uma dúzia ou mais deve resolver)
Montagem
Prepare a madeira, de forma que ela fique a mais lisa possível, retirando todas as farpas e os possíveis defeitos.
Numa das bordas de menor largura, fixe dois parafusos, e, no centro da borda oposta, o outro parafuso.
Passe cada uma das pontas da borrachinha pelos parafusos da extremidade que contém dois parafusos.
Amarre no centro do elástico um pedaço de linha.
Puxando a borrachinha pela linha, estique-a na direção do parafuso que está no centro da outra extremidade e enrole a linha nele,
para que fique preso e esticado. Não encoste a borrachinha no parafuso; deixe uma folga de mais ou menos um centímetro.
Coloque algo que sirva de projétil dentro do vértice em “V” formado pela borrachinha esticada.
Coloque os lápis sobre a mesa, um paralelo ao outro, formando uma espécie de caminho por onde o canhão deverá se deslocar
após o tiro.
Coloque o conjunto já montado sobre a esteira de lápis e, com o fósforo, queime a linha, sem que o palito ou você encoste no
experimento.
Comentário: o peso do canhão é importante para se observar um bom recuo. Portanto, escolha bem a madeira que vai servir de
base para o canhão.
Esquema geral de montagem:

FRENTE B
CA
SA
DE
TI
PO
S/
AR
Q
UI
FÍSICA

VO
DA
ED
IT
O
RA

Fonte: Unesp/Bauru. Projeto Experimentos de Física com Materiais do Dia a Dia.


Disponível em: <www2.fc.unesp.br/experimentosdefisica/mec03.htm>. Acesso em: 4 fev. 2015. Adaptado.

Conservação da quantidade de movimento 29


Veja, no Guia do Professor, as respostas da “Revisão”. As resoluções encontram-se no portal,

REVISÃO
em Resoluções e Gabaritos.

As resoluções dos exercícios encontram-se no portal, em Resoluções e Gabaritos.

1 (Mack-SP) Um automóvel P de 800 kg está parado em um (16) O impulso causado por uma força resultante sobre
semáforo quando é atingido por trás por outro carro Q um corpo é igual à variação de sua quantidade de
de 1 200 kg. Imediatamente após a colisão, os carros se movimento.
movem juntos com velocidade de 12 m/s. A velocidade
do automóvel Q, no momento em que colidiu com o P, 4 (UFTM-MG) Em um recente acidente de trânsito, uma
era no mínimo de: caminhonete de 1,6 tonelada, a 144 km/h, atingiu ou-
a) 36 km/h. tro veículo, em uma grave colisão frontal, e conseguiu
b) 45 km/h. parar somente a 25 metros de distância do abalroa-
c) 54 km/h. mento. A intensidade média da força resultante que
d) 72 km/h. agiu sobre a caminhonete, do ponto do impacto ao de
e) 90 km/h. paragem, foi, em newtons, igual a:
a) 51 200.
b) 52 100.
2 (UFPE) Um barco a vela parte do repouso, em linha reta,
c) 65 000.
m sob efeito de uma brisa constante. A força de atrito en-
Ene-6 d) 72 400.
C 4
H-2 tre a superfície do barco e a superfície da água é propor-
e) 75 000.
cional à velocidade instantânea v do barco em relação à
m
Ene-6
C 5 água, sendo dada por f 5 kv, em N, em que k 5 1,0 kg/s. 5 (Uerj) Em uma partida de tênis, após um saque, a bola,
H-2
O gráfico mostra a dependência da velocidade do bar- de massa aproximadamente igual a 0,06 kg, pode atin-
m co, em relação à água, em função da distância percorrida
Ene-5 gir o solo com uma velocidade de 60 m/s. Admitindo
C 7
H-1 pelo barco. Qual é o trabalho realizado pela força de atrito que a bola esteja em repouso no momento em que
nem
durante os primeiros 60 m? Dê a sua resposta em joules. a raquete colide contra ela, determine, no SI, as varia-
E -5
C 8
H-1 ções de sua quantidade de movimento e de sua ener-
v (m/s)
gia cinética.
2,50
6 (Uerj) Observe a tabela abaixo, que apresenta as massas
m
Ene-5
de alguns corpos em movimento uniforme.
C 7
1,25 H-1
Corpos Massa (kg) Velocidade (km/h)
m
Ene-5
C 8
H1
- Leopardo 120 60
0
0 20 40 60 80 100 120 140 160 180 x (m) Automóvel 1 100 70
Caminhão 3 600 20
3 (UEPG-PR) Considerando o teorema da impulsão, calcule
a soma das alternativas que forem corretas. Admita que um cofre de massa igual a 300 kg caia, a partir
(01) No gráfico da variação da quantidade de movimen- do repouso e em queda livre, de uma altura de 5 m. Con-
to contra o tempo (ΔQ 3 t), o coeficiente angular da sidere Q1, Q2, Q3 e Q4, respectivamente, as quantidades de
reta apresentada corresponde ao valor da massa do movimento do leopardo, do automóvel, do caminhão e
corpo sobre o qual a força F é aplicada. do cofre ao atingir o solo. As magnitudes dessas grande-
(02) Para um instante t 5 0, a quantidade de movimento zas obedecem relação indicada em:
a) Q1 , Q4 , Q2 , Q3.
de um corpo é nula.
b) Q4 , Q1 , Q2 , Q3.
(04) Se a resultante de um sistema de força que atua so-
c) Q1 , Q4 , Q3 , Q2.
bre um corpo em movimento for nula, a velocidade
d) Q4 , Q1 , Q3 , Q2.
do corpo poderá ser alterada se houver variação da
massa do corpo. 7 (IME-RJ) Um corpo de 300 g de massa é lançado de uma
(08) O impulso é uma grandeza vetorial e a sua direção e altura de 2,20 m em relação ao chão, como mostrado na
o seu sentido são os mesmos que os da força. figura a seguir.

30 Conservação da quantidade de movimento


60° de martelo. O martelo consiste de uma esfera metálica
v0 5 20 m/s presa a um cabo que possui uma alça na outra extremi-
dade para o atleta segurar. O atleta deve girar o martelo
em alta velocidade e soltar a alça, permitindo que a esfe-
2,20 m ra possa continuar seu movimento na direção tangente
à trajetória circular. Suponha que o atleta aprendiz es-
teja sobre uma plataforma e gire o martelo num círculo
horizontal de raio 2 m e a uma altura de 3,2 m do solo
no momento que faz o arremesso. A esfera cai no solo
O vetor velocidade inicial v0 tem módulo de 20 m/s e faz
a uma distância horizontal de 32 m do ponto onde foi
um ângulo de 60° com a vertical. O módulo do vetor dife-
arremessada. Despreze a resistência do ar. Considere a
rença entre o momento linear no instante do lançamento
massa da esfera igual a 4 kg e a aceleração gravitacional
e o momento linear no instante em que o objeto atinge
igual a 10 m/s2. Com base nessas informações, calcule:
o solo, em kg ? m/s, é:
a) a velocidade tangencial da esfera no instante em que
Dado: aceleração da gravidade: 10 m/s2. ela é arremessada.
a) 0,60.
b) a aceleração centrípeta sobre a esfera no momento
b) 1,80.
em que ela é solta.
c) 2,25.
c) a quantidade de movimento (momento linear) e a
d) 3,00.
energia cinética da esfera no instante em que ela é
e) 6,60.
lançada.
8 (ITA-SP) Uma sonda espacial de 1 000 kg, vista de um sis-
11 (UFF-RJ) Um bloco de massa m 5 0,20 kg repousa sobre
tema de referência inercial, encontra-se em repouso no
m
Ene-5 m um plano inclinado de um ângulo θ 5 37° em relação
C 7
espaço. Num determinado instante, seu propulsor é liga-
H-1 Ene-5
C 2
H-2 à horizontal. O bloco é subitamente impulsionado, para-
do e, durante o intervalo de tempo de 5 segundos, os ga-
lelamente ao plano, por uma marretada, parando após
ses são ejetados a uma velocidade constante, em relação m
Ene-5
C 7 percorrer uma distância ΔS 5 0,45 m a partir de sua posi-
à sonda, de 5 000 m/s. H-1
ção inicial, como mostra a figura. (Dados: cos 37° 5 0,80 e
sen 37° 5 0,60.)
1000 kg

20 m/s 5 000 m/s


ΔS

No fim desse processo, com a sonda movendo-se a 20 m/s, u


a massa aproximada de gases ejetados é:
a) 0,8 kg.
Sabendo que o coeficiente de atrito cinético entre o blo-
b) 4 kg.
co e o plano é mc 5 0,50 e que a aceleração da gravidade
c) 5 kg.
é g 5 10 m/s2, determine:
d) 20 kg.
a) o trabalho realizado pela força de atrito durante o des-
e) 25 kg.
locamento ΔS;
9 (Cefet-MG) Um objeto, deslocando-se com uma quan- b) o trabalho realizado pela força peso do bloco durante
tidade de movimento de 20 kg ? m/s, colide com um o deslocamento ΔS;
obstáculo durante 0,010 s e para. O valor médio da força c) a velocidade do bloco imediatamente após a marretada;
impulsiva que atua nesse objeto é, em newtons: d) o valor do impulso que a marreta imprime ao bloco.
a) 1,0 ? 10−1. FRENTE B
12 (UFTM-MG) Um pedreiro, ao mover sua colher, dá movi-
b) 2,0 ? 10−1.
c) 1,0 ? 103. mento na direção horizontal a uma porção de massa de
d) 2,0 ? 103. reboco, de 0,6 kg, que atinge perpendicularmente a pare-
FÍSICA

de com velocidade de 8 m/s. A interação com a parede é


10 (UFPR) Um adolescente, inspirado pelos jogos olímpicos inelástica e tem duração de 0,1 s.
no Brasil, está aprendendo a modalidade de arremesso No choque, a massa de reboco se espalha uniformemen-
te, cobrindo uma área de 20 cm2.

Conservação da quantidade de movimento 31


Nessas condições, a pressão média exercida pela massa b)
sobre os tijolos da parede é, em Pa:
ΔQ&
a) 64 000. d) 24 000. Q&final
b) 48 000. e) 16 000.
c) 36 000.
Q&inicial
13 (UFRN) O funcionamento de um gerador eólico é baseado
m na interação entre suas pás e o vento. Nessa interação, as c)
Ene-5
C 7
H1
- pás do gerador funcionam como defletor para a massa de ar
ΔQ&
m incidente. Durante a interação, o vetor quantidade de movi- Q&final
Ene-5
C 8
H-1 mento do ar incidente Q&inicial tem a orientação alterada para
quantidade de movimento do ar refletido, Q&final, pela presen-
ça das pás, conforme mostrado na figura abaixo. Q&inicial

d) Q&inicial

Q&final
ΔQ&

14 Um vagão vazio, sem cobertura, está se deslocando sobre


m trilhos retos horizontais, sem atrito.
Ene-5
C 8
H 1 a) Começa a chover e a água, caindo verticalmente, vai
-

se acumulando no interior do vagão. O módulo da ve-


locidade do vagão aumenta, diminui ou não se altera?
b) Para de chover e a água acumulada gradualmente es-
coa por um orifício existente no fundo do vagão. O
Corte transversal de módulo da velocidade do vagão aumenta, diminui ou
uma pá do gerador não se altera?
Q&inicial
15 Uma pessoa empurra um carrinho com uma força F&1
como mostra a figura deste exercício. No interior do car-
Q&final rinho, existe um pacote que comprime seu fundo com
uma força F&2. Sejam F&3 a força de reação do fundo do car-
ro sobre o pacote e F&4 a força de atrito do chão sobre suas
rodas e considerando o sistema constituído pelo carrinho
A variação da quantidade de movimento da massa de
e o pacote, responda:
ar incidente sobre as pás faz com que elas girem em
torno de seu eixo, gerando energia elétrica. Tal variação
AVITS ESTÚDIO GRÁFICO/ARQUIVO DA EDITORA

F&3 F&1
na quantidade de movimento do ar, ΔQ&, é expressa por
ΔQ% 5 Q&final 2 Q&inicial .
Neste sentido, a composição de vetores que melhor re-
presenta a variação da quantidade do movimento do ar
está representada por: F&4
a) Q&inicial

F&2
Q&final
ΔQ&
a) Quais dessas forças são internas?
b) Quais dessas forças são externas?

32 Conservação da quantidade de movimento


16 (UFSC) Dois patinadores, um homem e um menino, de a) Admitindo que o martelo caia de uma altura h sobre
massas respectivamente iguais a 60 kg e 30 kg, estão em a estaca, calcule o módulo da velocidade v adquirida
pé, de frente um para o outro, em repouso, sobre uma pelo conjunto martelo 1 estaca imediatamente após
superfície de gelo, lisa, plana e horizontal. Quando um o choque.
empurra o outro, o homem adquire uma velocidade de b) Calcule o valor da energia “perdida” durante o golpe
0,3 m/s em relação ao gelo. do martelo contra a estaca. Expresse resultado em ter-
mos de h, mm e me.

18 (IFSC) Frederico (massa 70 kg), um herói brasileiro, está


m de pé sobre o galho de uma árvore a 5 m acima do
Ene-5
C 7
H-1 chão. Segura um cipó que está preso em um outro
CASA DE TIPOS/ARQUIVO DA EDITORA

galho, que lhe permite oscilar, passando rente ao solo


sem tocá-lo.
Frederico observa um pequeno macaco (massa 10 kg)
no chão, que está preste a ser devorado por uma onça, o
maior felino da fauna brasileira. Desprezando a resistência
do ar para essa operação de salvamento, dê a soma da(s)
proposição(ões) corretas(s).
Considerando desprezível o atrito entre os patins dos Dado: considere Frederico e o macaco como partículas.
patinadores e o gelo, dê a soma da(s) proposição(ões) (01) Há conservação de energia mecânica do nosso he-
correta(s). rói, quando ele oscila do galho da árvore até o chão.
(01) A distância entre os patinadores 2,0 s após eles se (02) A velocidade do nosso herói, quando chega ao chão,
separarem é de 1,8 m. antes de pegar o macaco, é 10 m/s.
(02) A energia mecânica do sistema homem-menino se (04) O choque entre o nosso herói e o macaco é elástico.
conserva. (08) O choque entre o nosso herói e o macaco é perfeita-
(04) As forças que o homem e o menino fazem um sobre mente inelástico.
o outro são conservativas. (16) Imediatamente após pegar o macaco, a velocidade
(08) A força externa resultante sobre o sistema homem- do conjunto (nosso herói e macaco) é 10 m/s.
-menino é nula. (32) Para esta operação de salvamento, houve conserva-
(16) Como a massa do homem é maior do que a do me- ção da quantidade de movimento.
nino, a quantidade de movimento do sistema tem o
mesmo sentido que a quantidade de movimento do 19 (Cefet-MG) Uma bola branca de sinuca, com velocidade
m de 10 m/s na direção x e sentido positivo, colide elastica-
homem. Ene-5
C 7
H-1 mente, na origem do sistema de coordenadas xy, com uma
(32) As forças internas que atuam no sistema homem-
bola preta de mesma massa, inicialmente em repouso.
-menino não alteram a quantidade de movimento m
Ene-6
C 0
total do sistema. H-2 y

17 (UFJF-MG) Um bate-estaca é um dispositivo utilizado


m no início da construção de um edifício. Ele consiste em
Ene-5 v&FB
C 7
H-1 um mecanismo que provoca a queda de uma grande
massa (o martelo) sobre uma estaca para encravá-la no 30°
m
Ene-5
C 8 solo. Depois da queda, a estaca e o martelo se movem
H-1 60° x
juntos para baixo, sendo que o conjunto (martelo 1 FRENTE B
estaca) adquire, imediatamente após o impacto, uma
velocidade v. Supondo-se que o solo seja muito mole,
essa velocidade pode ser considerada constante du-
FÍSICA

rante certo intervalo de tempo e, durante esse interva-


lo, a energia e o momento transferidos ao solo podem
ser desprezados. As massas da estaca e do martelo são v&FP
me e mm, respectivamente.

Conservação da quantidade de movimento 33


Após a colisão, as velocidades finais das bolas preta, vFP , e bola incidente. Supondo que as bolas tenham massas
branca, vFB , são, respectivamente, em m/s, iguais a: idênticas, que o choque seja elástico e que a velocidade
a) 3,2 e 7,6. da bola incidente seja de 2 m/s, qual será, em m/s, a velo-
b) 3,5 e 5,8. cidade inicial da bola alvo após a colisão?
c) 5,0 e 8,7. a) 0,5
d) 6,0 e 4,5. b) 1
c) 2
20 (UFRGS-RS) Assinale a alternativa que preenche corre- d) 4
m tamente as lacunas da sentença abaixo, na ordem em e) 8
Ene-5
C 7
H-1 que aparecem.
m
Dois blocos, 1 e 2, de massas iguais, movem-se com 22 (UFSM-RS) A hipótese mais aceita nos meios científicos
Ene-5
C 8 velocidades constantes de módulos v . v , seguindo m atribui a grande extinção da fauna terrestre, ocorrida há
H-1 1i 2i Ene-5
C 7
a mesma direção orientada sobre uma superfície hori- H-1 aproximadamente 65 milhões de anos, à colisão de um
zontal sem atrito. Em certo momento, o bloco 1 colide corpo celeste de grandes dimensões, possivelmente um
m
Ene-5
com o bloco 2. A figura representa dois instantâneos C 8 cometa, com a superfície da Terra. Esse bólido foi absor-
H-1
desse movimento, tomados antes (X) e depois (Y) de vido pela Terra e o que seguiu foi um súbito desequilíbrio
o bloco 1 colidir com o bloco 2. A colisão ocorrida en- ambiental, que incluiu obstrução da passagem da luz so-
tre os instantes representados é tal que as velocidades lar, maremotos e violentas erupções vulcânicas.
finais dos blocos 1 e 2 são, respectivamente, v1f 5 v2i e A respeito das propriedades desse tipo de colisão, com-
v2f 5 v1i . plete as lacunas na afirmação a seguir.
v1i X v2i Trata-se de um exemplo de choque perfeitamente
, em que o movimento linear do sistema cometa-
-Terra conservado. Nesse evento, ocorre
1 2 da energia mecânica.
Assinale a sequência correta.
a) inelástico – é – conservação
b) elástico – não é – conservação
c) elástico – não é – dissipação
d) inelástico – não é – conservação
e) inelástico – é – dissipação
v1f Y v2f

23 (Fuvest-SP) Para realizar testes que mostrem a vanta-


gem do uso do cinto de segurança, dois carros idênti-
1 2
cos, A e B, vão colidir contra uma parede vertical rígida
com velocidades de módulo v0 5 20 m/s. No carro A,
utiliza-se um boneco com massa de 80 kg e usando
cinto de segurança, que o faz parar em 2,0 ? 10−1 s. No
carro B, utiliza-se um boneco com massa de 80 kg que
Com base nessa situação, podemos afirmar corretamente não está usando cinto de segurança e vai parar em um
que a colisão foi e que o módulo do impulso so- intervalo de tempo de 2,0 ? 10−3 s ao colidir contra o
bre o bloco 2 foi que o módulo do impulso sobre volante. Seja FA a intensidade da força média que fez
o bloco 1. o boneco A parar. Seja FB a intensidade da força média
a) inelástica – o mesmo que fez o boneco B parar. Considere as proposições a
b) inelástica – maior seguir:
c) perfeitamente elástica – maior I. O módulo da variação da quantidade de movimento
d) perfeitamente elástica – o mesmo dos dois bonecos foi o mesmo durante a freada e vale
e) perfeitamente elástica – menor 1,6 ? 103 kg ? m/s.
II. O módulo do impulso recebido pelos dois bonecos foi
21 (UPF-RS) Em uma mesa de sinuca, uma bola é lançada o mesmo durante a freada e vale 1,6 ? 103 N ? s.
frontalmente contra outra bola em repouso. Após a co- III. FA 5 FB 5 8,0 ? 103 N
lisão, a bola incidente para e a bola alvo (bola atingida) IV. FA 5 100; FB 5 8,0 ? 105 N
passa a se mover na mesma direção do movimento da V. FB 5 100; FA 5 8,0 ? 105 N

34 Conservação da quantidade de movimento


Estão corretas apenas: 25 (UFF-RJ) Duas partículas, de massas m1 e m2 , colidem fron-
a) I, II e V. m talmente. A velocidade de cada uma delas, em função do
Ene-6
C 4
b) I, II e IV. H2
- tempo, está representada no gráfico:
c) I e III. v (m/s)
m
Ene-6
d) II e IV. C 4
H-2
e) I e V. 7 m1
m 6
Ene-5 5
C 7
24 (ITA-SP) Um disco rígido de massa M e centro O pode H1
-
4
oscilar sem atrito num plano vertical em torno de uma 3
2
articulação P. 1 m2
0
21 1 2 3 4 5 6 7 8 9 10 t (s)
22
23
g& P g& P

u
A relação
O entre m1 e m2 é:
O
m a) m2 5 5m1.
b) m2 5 7m1.
v& 3m
c) m2 5 1 .
Projétil 1 disco 7
7m1
d) m2 5 .
3
e) m2 5 m1.
g& P g& P
26 (Fuvest-SP) Maria e Luísa, ambas de massa M, patinam
u m no gelo. Luísa vai ao encontro de Maria com velocidade
O Ene-5
C 9
O H-1 de módulo V. Maria, parada na pista, segura uma bola de
massa m e, num certo instante, joga a bola para Luísa. A
m
v& Ene-5
C 1 bola tem velocidade de módulo v, na mesma direção de
V&&. Depois que Luísa agarra a bola, as velocidades de Maria
H-2
Projétil 1 disco
e Luísa, em relação ao solo, são, respectivamente:
Dados: V e v são velocidades em relação ao solo; consi-
O disco é atingido por um projétil de massa m ,, M que dere positivas as velocidades para a direita; desconsidere
se move horizontalmente com velocidade v& no plano do efeitos dissipativos.
disco. Após a colisão, o projétil se incrusta no disco e o
CASA DE TIPOS/
ARQUIVO DA EDITORA

conjunto gira em torno de P até o ângulo θ. Nestas con-


dições, afirmam-se: v&
I. A quantidade de movimento do conjunto projétil 1 V&

disco se mantém a mesma imediatamente antes e


imediatamente depois da colisão.
II. A energia cinética do conjunto projétil 1 disco se
Maria Luísa
mantém a mesma imediatamente antes e imediata-
mente depois da colisão.
a) 0; v 2 V
III. A energia mecânica do conjunto projétil 1 disco ime-
V
diatamente após a colisão é igual à da posição de ân- b) 2v; v 1
2
gulo θ . FRENTE B
2 c) 2 mv MV
;
É (São) verdadeira(s) apenas a(s) assertiva(s): M m
a) I. 2 mv (mvv 2 MMV)
d) ;
FÍSICA

b) I e II. M (M 1 m)
c) I e III.  MV 2 mv   mv 2 MV 
d) II e III.  2   2 
e) ;
e) III. M (M 1 m)

Conservação da quantidade de movimento 35


27 (Fuvest-SP) Dois pequenos discos, de massas iguais, são e)
m lançados sobre uma superfície plana e horizontal, sem
Ene-5
C 7
H 1 atrito, com velocidades de módulos iguais. A figura a se-
-

guir registra a posição dos discos, vistos de cima, em in-


tervalos de tempo sucessivos e iguais, antes de colidirem,
próximo ao ponto P. P

28 (PUC-SP) Dois carros, A e B, de massas iguais, movem-


-se em uma estrada retilínea e horizontal, em sentidos
P opostos, com velocidades de mesmo módulo. Após se
chocarem frontalmente, ambos param imediatamente
devido à colisão.

AVITS ESTÚDIO GRÁFICO/


ARQUIVO DA EDITORA
A B
Dentre as possibilidades representadas, aquela que pode
corresponder às posições dos discos, em instantes suces-
sivos, após a colisão, é:
A B
a)

Pode-se afirmar que, no sistema, em relação à situação


descrita:
P a) há conservação da quantidade de movimento do sis-
tema e da sua energia cinética total.
b) não há conservação da quantidade de movimento do
b) sistema, mas a energia cinética total se conserva.
c) nem a quantidade de movimento do sistema nem a
P
energia cinética total se conservam.
d) a quantidade de movimento do sistema é transforma-
da em energia cinética.
e) há conservação da quantidade de movimento do sis-
tema, mas não da sua energia cinética total.

c) 29 (Unifesp) Com o auxílio de um estilingue, um garoto lan-


ça uma pedra de 150 g verticalmente para cima, a partir
do repouso, tentando acertar uma fruta no alto de uma
árvore. O experiente garoto estica os elásticos até que
P
estes se deformem de 20 cm e, então, solta a pedra, que
atinge a fruta com velocidade de 2 m/s.
Considerando que os elásticos deformados armazenam
energia potencial elástica de 30,3 J, que as forças de atrito
d) são desprezíveis e que g 5 10 m/s2, determine:
a) a distância percorrida pela pedra, do ponto em que é
solta até o ponto em que atinge a fruta;
b) o impulso da força elástica sobre a pedra.
P

30 (Mack-SP) A esfera A, de pequenas dimensões e massa


200 g, desliza com velocidade 5,00 m/s sobre a superfície
plana e horizontal, quando colide frontalmente com a es-

36 Conservação da quantidade de movimento


fera B, idêntica à A, inicialmente em repouso. A esfera B, Após a colisão:
suspensa por um fio ideal que é mantido tenso devido à
a) qual deve ser o valor da quantidade de movimento
ação de seu próprio peso, é tangente à superfície hori-
do sistema na direção MN? E na direção perpendi-
zontal, sem estar nela apoiada. Sabendo que o choque é
cular a MN?
perfeitamente elástico e que a aceleração gravitacional é
b) calcule, baseando sua resposta na questão a, o valor
g 5 10 m/s2, podemos afirmar que:
da velocidade da bola B.

33 (Fuvest-SP) Uma granada foi lançada verticalmente, a


partir do chão, em uma região plana. Ao atingir sua al-
tura máxima, 10 s após o lançamento, a granada explo-
3,00 m diu produzindo dois fragmentos, com massa total igual
A B
a 5 kg, lançados horizontalmente. Um dos fragmentos,
com massa igual a 2 kg, caiu a 300 m ao sul do ponto de
lançamento 10 s depois da explosão.

a) a esfera A para e a B se eleva no máximo 1,25 m.


b) a esfera A para e a B se eleva no máximo 0,625 m.
c) a esfera B permanece em repouso e a A retorna com g&
velocidade 5,0 m/s.
d) a esfera B se eleva 1,25 m e a A retorna com velocida-
de 5,0 m/s.
e) a esfera B se eleva 0,625 m e a A retorna com veloci-
dade 5,0 m/s. Pode-se afirmar que a parte da energia liberada na ex-
plosão, e transformada em energia cinética dos fragmen-
31 (Vunesp) Um corpo A, de massa m e velocidade v0, co- tos, é aproximadamente de:
m lide elasticamente com um corpo B em repouso e de
Ene-5
a) 900 J. c) 3 000 J. e) 9 000 J.
C 9
H-1 massa desconhecida. Após a colisão, a velocidade do b) 1 500 J. d) 6 000 J.
v
corpo A é 0 , na mesma direção e no mesmo sentido
2 34 (Mack-SP) Um pequeno corpo C1, de massa 2 kg, é aban-
que os do corpo B. A massa do corpo B é: m
Ene-5
C 7
donado do repouso no ponto A do trilho representado
a) m .
H-1 abaixo. Sem perder o contato com o trilho, esse corpo
3 m desliza sem atrito até atingir o ponto B, num trecho ho-
Ene-6
C 0
b) m .
H-2 rizontal, quando se choca frontalmente com um outro
2 corpo C2, de massa 3 kg, inicialmente em repouso.
c) 2m.
A
d) 3m.
e) 6m.

32 Uma bola A, de massa 2,0 kg, move-se sobre uma mesa C


1,25 m
m lisa e horizontal, ao longo da reta MN (veja figura) com
Ene-5
C 7
H1
- uma velocidade de 2,0 m/s. Ela colide obliquamente 0,35 m
m
com uma bola B, de massa 10,0 kg, inicialmente em re- B
Ene-6
C 0 pouso. Observa-se que, após a colisão, a bola A move-se
H-2
em uma direção perpendicular a MN, como mostra a fi-
Sabendo que o choque é perfeitamente elástico, o
gura, com uma velocidade de 1,5 m/s.
segundo corpo atingirá o ponto C do trilho com ve- FRENTE B
locidade de:
AVITS ESTÚDIO GRÁFICO/
ARQUIVO DA EDITORA

a) 9,0 m/s.
A b) 6,0 m/s.
FÍSICA

M A N c) 5,0 m/s.
B
d) 4,0 m/s.
B e) 3,0 m/s.

Conservação da quantidade de movimento 37


REFERÊNCIAS BIBLIOGRÁFICAS
ALONSO, M.; FINN, E. J. Física. Madrid: Addison-Wesley, 1999.
CAMINHOS DA CIÊNCIA (Coleção). São Paulo: Scipione, 1996. 8 v.
CANIATO, R. As linguagens da Física Mecânica. São Paulo: Ática, 1990.
CHAVES, A. Física. Rio de Janeiro: Reichmann & Affonso, 2000. v. 1-4.
FEYNMANN, R.; LEIGHTON, R. B.; SANDS, M. Lectures on Physics. New York: Addison-Wesley, 1972.
GRUPO de reelaboração do ensino de Física (Gref). São Paulo: Edusp, 1990. v. 1.
LUCIE, P. Física básica: Mecânica. Rio de Janeiro: Campus, 1979. v. 2-3.
NUSSENZVEIG, M. H. Curso de Física básica. São Paulo: Edgard Blücher, 1988. v. 1-2.
OKUNO, E.; DUARTE, M. Física do futebol. 1. ed. São Paulo: Oficina de Textos, 2012.
REIF, F. Berkeley Physics course: statistical Physics. Berkeley: McGraw-Hill Co., 1965. v. 5.
TIPLER, P. A. Física. Rio de Janeiro: Guanabara Dois, 1978.

38 Conservação da quantidade de movimento


MAIS ENEM
Ciências Humanas e suas Tecnologias
Ciências da Natureza e suas Tecnologias
Linguagens, Códigos e suas Tecnologias
Matemática e suas Tecnologias

Um dos grandes problemas de saúde pública no Brasil é o consumo legalizado de álcool etílico. De acordo com pesquisas acadêmicas,
doenças provocadas exclusivamente pelo alcoolismo, invalidez e morte precoce em decorrência do abuso de bebidas alcoólicas, além dos efeitos
fisiológicos do álcool, geram gastos aos cofres públicos estimados em 8 bilhões de reais ao ano. Além desses fatores, o consumo de álcool associado
ao ato de dirigir tem sido motivo de preocupação para as autoridades brasileiras. Durante o período de 2000 a 2010, tivemos aumento de 41,4%
no número de mortes nas vias públicas, passando de 28 995 para 40 989. Observe na figura 1 a significativa redução de óbitos de pedestres
por acidentes de trânsito e o aumento expressivo no número de mortos em acidentes envolvendo automóveis e motocicletas. Estima-se que
quase 50% dos acidentes de trânsito deve-se à ingestão de álcool durante a condução de veículos. A própria legislação brasileira dificultava a
redução do número de acidentes de trânsito causados pelos motoristas alcoolizados, pois o Superior Tribunal de Justiça havia decidido que o
bafômetro e o exame de sangue eram obrigatórios para comprovar o crime. O bafômetro é um aparelho que permite determinar a concentração
de bebida alcoólica analisando o ar exalado dos pulmões de uma
pessoa. É também conhecido pela denominação técnica “etilômetro”, 18
Pedestre
devido às reações que envolvem o álcool etílico presente na baforada 16
Motociclista
do suspeito e um reagente. Com isso, os motoristas recusavam-se a 14
soprar o bafômetro quando surpreendidos em blitz policiais, valendo-se Automóvel
12
do direito constitucional que diz que “ninguém é obrigado a produzir
provas contra si mesmo”. Neste caso, o condutor alcoolizado apenas era 10 % dos óbitos
multado, tinha seu veículo apreendido e perdia a carteira de habilitação, 8
mas ficava livre de qualquer acusação criminal. Isso acontecia porque a 6
lei previa como conduta proibida dirigir com mais de 6 dg/L (decigramas
4
por litro) de álcool no sangue. Em 2013, entrou em vigor a nova lei,
em que passa a ser crime “conduzir veículo automotor com capacidade 2
psicomotora alterada em razão da influência de álcool ou de outra 0
substância psicoativa que determine dependência”. Assim, o limite de 1996 1998 2000 2002 2004 2006 2008 2010
álcool passou a ser uma das formas de se comprovar a embriaguez, e
Fig. 1 – Taxas de óbitos (em 100 mil habitantes) nos acidentes de trânsito por
não mais um requisito de punição. categoria. Brasil. 1996/2010 (2010: dados preliminares).
O passo a passo do funcionamento do bafômetro
1. Com a ajuda de um catalisador, o álcool expirado reage com o oxigênio presente no aparelho.
2. A reação libera ácido acético, íons de hidrogênio e elétrons.
3. Os elétrons passam por um fio condutor, gerando corrente elétrica. Quanto mais álcool, maior a corrente: um microchip “traduz” o
impulso elétrico e dá a concentração de álcool no sangue.
4. Ao fim do processo, sobra só água na forma de vapor. A concentração é mostrada no visor e qualquer resultado acima de zero é
considerado acima do permitido.
Com base em: INSTITUTO SANGARI. Mapa da violência 2012: acidentes de trânsito. São Paulo, abr. 2012.
Disponível em: <http://vias-seguras.com/documentacao/arquivos/mapa_da_violencia_2012_acidentes_de_transito>. MOLINI, Ana Maria.
Aula experimental como instrumento pedagógico no ensino de Química. Secretaria da Educação do Governo do Estado do Paraná.
Disponível em: <www.diaadiaeducacao.pr.gov.br/portals/cadernospde/pdebusca/producoes_pde/2009_uel_quimica_artigo_ana_maria_molini.pdf>.
Lei n. 9 503, de 23 de setembro de 1997. Disponível em: <www.planalto.gov.br/ccivil_03/leis/l9503.htm>. Acesso em: 13 jan. 2015.

1 Segundo a polícia, um condutor dirigia um veículo de 0,8 tonelada a 100 km/h quando bateu de frente com outro veículo de 1,0 tonelada
que trafegava a 53 km/h. Ambos os condutores sofreram ferimentos leves e foram levados ao hospital. Após submetê-los ao teste do ba-
fômetro, a polícia constatou em um deles o teor alcoólico de 0,99 mg/L.
Um vídeo gravado no local mostra, momentos após a colisão frontal, os dois veículos engavetados se movendo como um único corpo
por um curto intervalo de tempo. Desprezando-se o atrito entre os veículos e a pista, o módulo da velocidade dos carros unidos após
a colisão, em m/s, foi de aproximadamente: b
a) 15 m/s na contramão. d) 15 m/s na mão correta.
b) 4 m/s na contramão. e) Os carros engavetados permanecem parados após a colisão.
c) 4 m/s na mão correta.

39
QUADRO DE IDEIAS

Conservação da
quantidade de
movimento Vice-presidência: Mário Ghio Júnior
Direção: Carlos Roberto Piatto
Direção editorial: Lidiane Vivaldini Olo
Conselho editorial: Bárbara Muneratti de Souza Alves,
Carlos Roberto Piatto, Daniel Augusto Ferraz Leite,
Eduardo dos Santos, Eliane Vilela, Helena Serebrinic,
Lidiane Vivaldini Olo, Luís Ricardo Arruda de Andrade,
Impulso Marcelo Mirabelli, Marcus Bruno Moura Fahel,

I & 5 F& ? Δt
Marisa Sodero, Ricardo Leite, Ricardo de Gan Braga,
Tania Fontolan
Gerência editorial: Bárbara Muneratti de Souza Alves
Coordenação editorial: Adriana Gabriel Cerello
Edição: Tatiana Leite Nunes (coord.), Pietro Ferrari
Assistência editorial: Carolina Domeniche Romagna,
Quantidade de movimento Rodolfo Correia Marinho
Conservação de movimento
Colisões Revisão: Hélia de Jesus Gonsaga (ger.),
de um sistema de partículas q 5 mv Danielle Modesto, Edilson Moura, Letícia Pieroni,
Marília Lima, Tatiane Godoy, Tayra Alfonso,
Vanessa Lucena; Colaboração: Aparecida Maffei,
Thaise Rodrigues
Coordenação de produção:
Relação entre impulso e Fabiana Manna da Silva (coord.);
Colaboração: Adjane Oliveira, Dandara Bessa
Diretas e oblíquas quantidade de movimento Supervisão de arte e produção: Ricardo de Gan Braga
I & 5 Δq& 5q&2 2 q&1 Edição de arte: Daniela Amaral
Diagramação: Antonio Cesar Decarli,
Elásticas e Claudio Alves dos Santos, Fernando Afonso do Carmo,
inelásticas Flávio Gomes Duarte, Kleber de Messas
Iconografia: Sílvio Kligin (supervisão),
Marcella Doratioto; Colaboração: Fábio Matsuura,
Fernanda Siwiec, Fernando Vivaldini
Licenças e autorizações: Edson Carnevale
Capa: Daniel Hisashi Aoki
Foto de capa: Fábio Colombini
Projeto gráfico de miolo: Daniel Hisashi Aoki
Editoração eletrônica: Casa de Tipos

Todos os direitos reservados por SOMOS Educação S.A.


Avenida das Nações Unidas, 7221
Pinheiros – São Paulo – SP
CEP: 05425-902
(0xx11) 4383-8000
© SOMOS Sistema de Ensino S.A.
Dados Internacionais de Catalogação na Publicação (CIP)
(Câmara Brasileira do Livro, SP, Brasil)

Luz, Antônio Máximo Ribeiro da


Sistema de ensino ser : ensino médio, caderno
5 : física : frente B : PR / Antônio Máximo
Ribeiro da Luz, Beatriz Alvarenga Álvares. --
2. ed. -- São Paulo : Ática, 2015.

1. Física (Ensino médio) I. Álvares, Beatriz


Alvarenga. II. Título

15-01591 CDD-530.07

Índices para catálogo sistemático:


1. Física : Ensino médio 530.07

2015
ISBN 978 85 08 17316-7 (AL)
ISBN 978 85 08 17313-6 (PR)
1ª edição
1ª impressão

Impressão e acabamento

Uma publicação

Você também pode gostar